Anda di halaman 1dari 109

CHAPTER - 1 DEFINITION BASED QUESTIONS

In this BRM we will deal with Verbal Reasoning. The first chapter deals with 'definition based' ' questions. These questions are based on certain given definition/s. Students often get puzzled with bombastic terms like 'implicit statements', 'assertive statements', 'supporting reason', 'counter-argument', 'irrelevant argument', 'facts, inference and judgements', 'definitely true, probably true, definitely false, probably false'. We will try to deal with some of these definitions with the help of the following passage. .

PASSAGE Children in families that habitually watch television during meals eat fewer fruits and vegetables than those who don't, and consume more Pizza, snack food and caffeine laced soft drinks, U.S. researchers reported.

The report from the Tufts University in Boston was based on a study on the eating habits of 91 families in neighbourhoods adjacent to Washington, most of them in Maryland.

Katherine Coon, lead author of the study, said that a number of factors might be at work linking eating habits to watching television, but she believed TV itself - and the kinds of food advertised heavily on it - might be a powerful influence.

Implicit Statements : Implicit statements are those that are not stated in the passage but are implied or suggested. The statement "There are other researchers involved in the study" - is implied in the passage as Katherine is the lead author (not the only researcher).

Assertions : Assertions are claims, declarations or affirmations. For example, the statement "Children in families that habitually watch television during meals eat fewer fruits and vegetables than those who don't, and consume more Pizza, snack food and caffeine laced soft drinks, U.S. researchers reported" - is an assertion.

Supporting Reason : That which supports the assertion or foregrounds the point expressed in the assertion. The statement "TV itself and the kind of food advertised heavily on it might be a powerful influence" - supports the earlier statement and therefore qualifies as a supporting reason.

Counter Argument : A counter argument is that which states the opposite of what is stated in the passage or runs contrary to the assertion. The statement, " Eating and watching TV does not have any connection whatsoever", is a counter argument.

Irrelevant Argument : These are neither stated nor implied but are totally unconnected to the subject under discussion. The statement, "Katherine Coon has been a child psychologist for 15 years now" is an irrelevant argument. Note that nowhere is it related to the assertion given. Besides these there are a few more definition based questions which will be dealt with individually. Note : Always remember to read the definitions carefully before attempting the questions.

1. Argument, Assertion and Counter-argument: An Argument is a statement meant to convince another person about your point of view. An Assertion is a point of view. A Counter-argument contains logic opposing the assertion. Based on the above definitions, classify each of the given definitions into: I] Assertion III] Counter argument Example I a] One look at the Mumbai roads and you know why traffic moves so slow. II] Supporting Reason IV] Irrelevant argument

b] The road surface and the potholes following the monsoons result in traffic getting held up for hours. c] Mumbai has always been the financial capital of the city.

d] Mumbai has one of the best road surfaces in the country where you can easily drive at 70 kms per hr within the city. 1] I II III IV 2] I II IV III 3] II I III IV 4] IV I II IV

Solution 1. 2. Read the definitions carefully. Read the given statements and try to find the assertion. For example : The assertion here is [a] which states that in Mumbai traffic moves slow. 3. Once you've spotted the assertion look for a counter argument - it is the easiest to spot. However, remember that there may be questions without a counter argument. Here [d] is the counter argument as it nullifies the claim that Mumbai has bad roads which cause traffic to slow down. 4. [b] supports the claim by pointing out that the bad road surface along with potholes slow down traffic. 5. [c] is irrelevant to the argument as the topic under discussion is "slow traffic and not even remotely connected to the fact that Mumbai is the 'financial capital of India.' Important : You can also look at the options after fixing the Assertion and the Counter argument/ Supporting Reason, in this example there is only one option with I beginning the sequence and III ending the sequence. Thus, [2] is the answer.

EXERCISE-1 DIRECTIONS for questions 1 to 5: An Argument is a statement meant to convince another person, about your point of view. An Assertion is a point of view. A Counter-argument contains logic opposing the assertion. Based on the above definitions, classify each of the given sets of statements into: I] Assertion II] Counter argument 1. II] Supporting reason IV] Irrelevant argument

a] Those who are slow to react to changes are often swept away by them. b] Often, changes are fads, mere whims of fashion. c] Haste makes waste. d] When events unfold quickly, you have to be fast on your feet. 1] I, II, III, III 2] III, IV, II, I 3] II, IV, III, I 4] I, IV, I, II

2.

a] The cat is not an affectionate animal. b] It does not require the emotional warmth that a dog does. c] It is affectionate; but not very demonstrative.

3.

a] Kerala is God's own country. b] It has more than its fair share of natural beauty. c] The people there cook most of their food in coconut oil. d] A lot of the people use coconut oil for their hair, too. 1] I, II, II, II 2] II, I, II, IV 3] I, II, IV, IV 4] I, II, III, I

4.

a] Individuals who seek solutions come up against a wall of indifference. b] There seems to be absolutely no one who cares. c] The inefficiency of public-sector behemoths is legendary. d] The 'Voluntary Retirement Scheme' is very popular here. 1] II, II, I, IV 2] I, I, IV, IV 3] II, II, IV, I 4] I, III, IV, IV

5.

a] Anger is ineffective. b] It is better to be feared than to be liked. c] Fear is a poor motivator. d] Fear is dysfunctional. 1] I, III, II, II 2] II, I, III, III 3] I, II, III, IV 4] IV, I, III, III

ANSWERS KEY

13

22

33

41

5-4

EXPLANATORY ANSWERS

EXERCISE-1
1. In this question, we see that [b] is irrelevant to the other statements. That, however, does not help us much as only option [1] is eliminated. A closer

examination shows that [d] is the Assertion, for which [a] is the Supporting reason and [c] the Counter Argument. Hence, [3] is the correct option.

2.

Going through the statements", [d] is clearly an irrelevant statement. The only

option that has this is [2]. Hence this is the correct answer - we do not have to go further. Clearly, elimination is an important tool to be used in definitionbased questions. Hence, [2].

3.

Here, we have two separate sets of statements - [a] and [b], which describe Kerala, and [c] and [d] which describe the uses of coconut oil. Clearly, the choice of which of these sets are relevant, will render the other set irrelevant. How do we choose? Look at statement [c]. It talks about "people there cook etc." There is no subject described beforehand, we do not know where "there" is. It is, therefore, not an independent statement. Nor is [d]. However, statement [a], "Kerala is God's own country" is an independent statement, and can be classified as an Assertion, [b] then becomes its Supporting Reason. Then, [c] and [d] become irrelevant to the argument. The correct option therefore, is [3].

4.

Again, [a] is irrelevant. That does not help us much, though, it only eliminates one option, [3]. A closer look reveals that [c] is the Assertion, with [a] and [b] being Supporting Reasons. Hence, [1],

5.

Statement [a] is irrelevant to the other statements which straightway gives us option [4]. Statement [b] is the Assertion, [c] and [d] are the Counter Arguments.

2. Conclusions and Implicit Statements :

For the questions below a small passage is followed by three conclusions. You have to identify whether the statements are implicit from in the main passage. Example 1 Sometimes plants lure ants with nectar and other foods. Ectatomma tuberculatum ants in Peru drink a sweet sap that oozes from green spots along the petal margins of passion flowers blooms. With open jaws, workers aid the plant by keeping pests from invading the flower. a] Only passion flowers are attacked by invaders.

b]

Ectatomma tuberculatum ants and passion flower plants have a symbiotic relationship.

c]

When attacked by invaders passion flower plants become completely defenseless.

1] only I is implicit 3] I and III are implicit

2] only II is implicit 4] none is implicit

Solution 1. Here again it is essential that you read the passage carefully to get the essence of the passage. The essence of the passage is : A particular species of ants feeds on the nectar of passion flowers and in return protects the plants from invaders.

2.

Read the conclusions and try to find out whether they are implied in the passage.

3.

We cannot say I is implicit because there is no data to prove that 'only' passion flowers are invaded.

4.

The passage clearly describes a symbiotic relationship, passion flowers provide food for ants while ants protect the plant from being invaded. Thus II is implied in the passage.

5.

Look at III, the passage does not hint at the plant's getting defenseless when attacked by invaders, therefore III is not implied. Thus [2] is the answer.

Example 2 On the island of Santiago, feral pigs - introduced in the 19th century by mariners so that they could have pork chops - have grown in number and appetite, and they now eat many of the eggs and hatchlings of the resident green sea turtles. On some islands the turtle's survival rate has declined dramatically. a] b] c] The green sea turtles on the islands in and around Santiago are endangered. Feral Pigs in Santiago are affecting the ecological balance. Before the 19th century feral pigs did not exist in Santiago. 2] I and II are implied 4] all are implied

1] only I is implied 3] II and III are implied

Solution 1. If you read the passage carefully you will see that the essence of the passage is : Feral Pigs introduced in Santiago in the 19th century are now posing a threat to the survival of the green sea turtles.

2.

Now read the conclusions. I is certainly implied, if the turtles' survival rate has declined then it must be endangered.

3.

II is implied because it is clearly the multiplication of the feral pigs and their liking for turtle eggs which is putting the survival rate of turtles at stake.

4.

Though not stated in so many words, the fact that feral pigs were 'introduced' by the mariners in the 19th century implies that they did not exist in Santiago before the 19th century. Thus III is implied as well. The answer therefore is [4].

EXERCISE-2

DIRECTIONS for questions 1 to 5: There is a statement followed by three assumptions numbered I, II, and III] An assumption is something supposed or taken for granted. You have to decide which of the assumptions are implicit in the statement, and mark the answer. 1. Picasso was, arguably, the most prominent exponent of the school of art which he practised. I] II] Picasso used to work in a particular style. There were other artists who could lay claim to the title of most famous exponent of the school of art which Picasso practiced. III] 1] 3] Picasso was a student. Only I is implicit. I and II are implicit. 2] Only II is implicit. 4] Only III is implicit.

2.

Love is, perhaps, more important for life than food and drink, because it gives us a reason for the other two.

I] II] III] 1] 3]

Food and drink are important for life. Love is our most important need in life. Those without life will neither eat nor drink. None of the three are implicit. I and II are implicit. 2] Only I is implicit. 4] Only II is implicit.

3.

"The refineries will, once more, exceed their production targets, ensuring fuel for all", said the relieved Petroleum Minister. I] II] Refineries have exceeded their production targets in the past. Whether refineries would exceed their production targets, was a matter of worry to the Petroleum Minister. III] 1] 3] The entire output of refineries is used as fuel. Only I and II are implicit. Only I is implicit. 2] Only III is implicit. 4] Only II is implicit.

4.

Mr. Patil is now in charge of this hotel, so, obviously, we are going to stop serving liquor. I] II] III] 1] 3] Mr. Patil used to like liquor earlier, but has now given it up. The hotel used to serve liquor before Mr. Patil took charge. Mr. Patil disapproves of imbibing alcohol. Only III is implicit. II and III are implicit 2] All are implicit. 4] None of the assumptions are implicit.

5.

Casablanca apart from being one of Bogart's most compelling performances, was also an outstanding film. I] II] III] 1] 3] Bogart was an actor. Bogart gave many compelling performances. Casablanca was a film about Bogart. All are implicit. Only I is implicit. 2] I and II are Implicit. 3] I and III are implicit.

ANSWERS KEY

13

22

31

43

5-2

EXPLANATORY ANSWERS

EXERCISE-2

1.

I is clearly implicit. II is implicit from the word 'arguably' used in the sentence. However, III is not at all implicit. The answer is, therefore [3].

2.

As 'Love is perhaps, more important....,' that food and drink are important, is implicit. Neither II nor III are implicit: therefore, option [2] is the answer.

3.

'once more' suggests that they had exceeded their production rates earlier, thus I is implicit, 'relieved' suggests that the minister was worried. Thus II is implicit as well. Hence, [1].

4.

I is not implicit at all.

II is implicit from 'We are going to stop...', and III is Answer is option [3].

clearly implicit from the entire sentence.

5.

'Casablanca' was a film and Bogart was a performer in it - therefore, the fact that he was an actor is implicit. Also, as it was one of his most compelling... makes implicit the statement that he gave many compelling performances. III, however is not implicit. Therefore, option [2] is the answer.

3. Fact, Inference, Judgement : A Fact is something which can be seen or heard, and is capable of verification. An Inference is a statement drawn or following from a Fact. A Judgement is an opinion, and implies approval or disapproval. Based on the above definitions, mark the following as Fact, Inference or Judgement. Let us take an example:

Example 1 a] She looks good in blue.

b] c] 1]

The population is growing at an alarming rate. The doctor prescribed some medicines for her. IJF 2] JJJ 3] JFJ 4] JIF

It is easiest to spot a Judgement. More often than not a Judgement has an 'adjective' and expresses a personal view.

Solution Read the definitions carefully 1. Look at statement [a], it is a personal opinion, to some she may not look good in blue. Thus it is a Judgement.

2.

Look at statement [b], there must be some record of population growth with which the present rate it is being compared, thus it is likely to be an Inference. However if you are not too sure, move on to the third statement.

3.

Statement [c] can be easily verified, you can see whether the doctor has prescribed medicine or not, it is therefore a Fact. Thus [4] is the answer.

Note : Even if you are not confident about the second statement, it shouldn't stop you from getting the correct answer. Since you are sure that the first statement is a Judgement and the last one is a fact look for an option which gives you such a combination. Only [4] gives you the combination therefore you can mark it off as the answer.

Example 2 a] Mrs. Rowling's fourth book, Harry Potter and the Goblet of Fire deals with magic and the supernatural. b] But more novel by far is another recent publication, Lost for Words, a story without words. c] 1] There is no historical .precedent for Mrs. Rowling's achievement. IJF 2] FJI 3] FJJ 4] IFI

Solution 1. Read all the statements and try to find out the Judgement first. Statement II is a Judgement as it expresses a personal opinion. To some, it may be a novel

attempt, to others it may not appear to be so. There are three options with J in the middle. So you can only rule out option [4].

2. Now look at statement I whether it deals with magic and the supernatural can be found out by just reading the book, thus it is a Fact. You can again look at the options to eliminate them. Here option [1] gets eliminated as it does not start with F.

3. Look at statement III, it can clearly not be a Judgement, someone must have checked up the records to see whether such a book was written or not, it is therefore drawn from some facts and is therefore an Inference. Thus, the answer is [2].

EXERCISE-3

DIRECTIONS for questions 1 to 5: A Fact is something which can be seen or heard, and is capable of being verified. An Inference is a statement drawn or following from a Fact. A Judgment is an opinion, and implies approval or disapproval. Using the above definitions, classify the set of statements below into Fact, Inference or Judgment.

1.

a) This is a black book. b) All black books are unlucky. c) Black books are more attractive than brown ones. 1]FFF 2]IJJ 3]JFJ 4] FJJ

2.

a) The man is on the grass. b) The grass is blue. c) Plastic is not a bio-degradable substance. 1] J J J 2] F F F 3] F I J 4] F F J

3.

a) It is difficult to type standing up. b) Gorillas prefer eating fruit to eating nuts.

c) The young chap was a hideous shade of red. 1] J I J 2] F F I 3] F F F 4] J J F

4.

a) She was typing, sitting at the desk. b) What I love about him is, his calm nature. c) The receptionist gave me a friendly smile. 1] J J J 2] F J J 3] F I I 4] F F I

5.

a) The stained glass ceiling depicts champagne on ice. b) All in all it's just another brick in the wall. c) We are the Sultans of Swing. 1] J J J 2] F I I 3] F J J 4] F F F

ANSWERS KEY

14

22

31

42

5-3

EXPLANATORY ANSWERS

EXERCISE-3
F-I-J questions can have 4 statements as well. The statements can be related to each other, or could be completely unrelated. A technique that is used very often in these kinds of questions is, 'elimination'. That is, using those definitions that you are sure of, in this way you can eliminate various options and arrive at the right one. In any kind of definition-based question, there will be some definitions which are less ambiguous than others. In the Fact-Inference-Judgment kind of questions, it is easiest to recognize a Judgment followed by Fact. An Inference, however, is a bit difficult to gauge and unless absolutely necessary, do not spend time debating whether a

statement is an Inference or not. This is explained in detail, as follows:

1.

Statement (a) is a Fact, for sure. Scanning the options, we see that, this eliminates options [2] and [3]. Also, to decide between options [1] and [4], all we need to do is decide whether statement (b) is a Fact or a Judgment. Of the two, it is definitely a Judgment. Hence, without further ado, we mark option [4]. Please note that we have not even looked at statement (c) in this case, we do not need to, to arrive at the correct option. You do not need to classify each statement, to select the correct option: just use the technique of elimination.

2.

In this question, on deciding that the first statement is a Fact, you are still left with [2], [3], and [4]: also, you will have to classify statements (b) and (c), there is no getting away from it. Statement (b): 'The grass is blue', is definitely a Fact. So that leaves us with options [2] and [4], i.e., we have to decide if the last statement is a Fact or a Judgment - you need not check if it's an Inference, as that option is not available. 'Plastic is not a bio-degradable substance' does not classify as a judgment: that leaves us with option [2] as the correct one Please note that, statements do not need to conform to the world as you know it; 'the grass is blue' therefore, need not confuse you. Just match the statement with the definition and classify it accordingly.

3.

Scanning the three statements, we see that statement (c) is the easiest to classify, as a Judgment. Well, there is only one option wherein the last statement is classified as a Judgment: Option [1].

4.

The first statement is easily classified as a Fact.

Thereafter, we see that the

second statement is all we need to classify; it is a Judgment, expressing an opinion. Once that is done, option [2] is clear.

5.

Again, classifying the first statement as a Fact leaves us with the need for classifying statement (b) or (c). Marking statement (b) as a Judgment, leaves us with option [3] as the correct one.

4. Probably True, Definitely True

A passage is followed by certain inferences. Mark [1] Definitely true if the inference follows directly from the data given in the passage. Mark [2] Probably true if the inference is probably but not definitely true. Mark (3J Probably false if the inference is probably but not definitely false. Mark [4] Definitely false if the inference contradicts the data in the passage.

Example 1 Jeans are on the comeback trail. Barely a few months ago, the big story was about denim's decline and the shift in the youth wardrobe collection in favour of an assortment of other fabric and styles, notably khakis and cargoes. But the tide's turning again. Denim's very much on the revival path, though not in the same old fashion. No, the body hugging tapered, faded, jaded and tough look is not the only kind which is carrying good old jeans forward. Softer feels, lesser weights, darker colours and crisper looks and yes even flares and embroidered borders are now very much in. 1. Jeans with embroidered borders are yet to be accepted.

2. Fashion moves in a cyclical manner.

3. Jeans had fallen out of favour with the youngsters.

4. The crisper look of Jeans may lose out to the rugged tough look of the good old denim.

Solution

1.

Read the passage carefully, taking in the essence of the passage.

2.

Look at the inferences one by one. [1] says that the embroidered jeans are 'yet to be accepted', whereas the last sentence of the passage says they are 'very much in', it is clearly a contradiction, therefore it should be marked as 'definitely false

- [4]'.

3.

Read inference [2] carefully, we have been given an instance of how jeans dominated the fashion scene to be taken over by khakis and how jeans is again on its path of revival. Thus the inference appears to be true. However we cannot say that it is definitely true on the basis of a single instance, it is best to put it as 'probably true - [2]'.

4.

Look at inference [3]. Read the second sentence of the passage, the inference is 'definitely true - [1]'.

5.

Look at inference [4], what may happen cannot be ascertained. However, in the light of the passage where crisper look is in, we cannot conclude that it will lose out to the rugged look. Thus, we mark it as 'probably false - [3]'.

Note : Many students get confused with the concept of 'probably true and probably false'. However, they are not the same. A probably true inference appears to be true in the light of the passage but is not stated in the passage and therefore cannot be ascertained. Similarly a probably false inference is something which appears to be false in the light of the passage, yet cannot be ascertained as false.

EXERCISE-4

DIRECTIONS for questions 1 to 4: Below, there are inferences drawn from the main passage. Mark [1] Definitely true if the inference follows directly from the data in the passage. Mark [2] Probably true if the inference is probably but not definitely true. Mark [3] Probably false if the inference is probably but not definitely false. Mark [4] Definitely false if the inference contradicts the data in the passage.

R. Spitz has found that, infants deprived of handling over a long period, will tend at length to sink into an irreversible decline and are prone to succumb eventually to disease. In effect this means that, what he calls emotional deprivation can have a fatal

outcome. These observations give rise to the idea of 'stimulus-hunger' and indicate that the most favoured forms of stimuli are those provided by physical intimacy, a conclusion not hard to accept on the basis of everyday experience.

An allied phenomenon is seen in grown-ups subjected to sensory deprivation - in the past, social and sensory deprivation is noted to have produced temporary mental disturbances in individuals condemned to long periods of solitary imprisonment. 1. 'Stimulus - hunger' leads to death.

2.

Only infants, deprived of physical intimacy, are affected adversely.

3.

Physical intimacy is an intrinsic necessity for the mental and physical health of humans.

4.

In everyday social interaction, some form of physical intimacy exists.

5.

Solitary confinement may turn a person into a lunatic.

ANSWERS KEY

11

24

32

42

5-2

EXPLANATORY ANSWERS

EXERCISE-4
1. The author says that emotional hunger can have a fatal outcome. Thus, it is safe to mark the inference as 'probably true'. Hence, [2].

2.

This statement is a definite contradiction of the data given in the paragraph.

Hence, [4], Definitely false.

3.

This statement is 'Probably true'. Why not 'Definitely true?' - because, the passage, while stating that physical stimulus certainly does contribute to the well-being of humans, does not definitely state that it is a necessity. Hence, [2].

4.

"A conclusion not hard to accept on the basis of everyday experience" - this phrase is what leads us to mark [2], 'Probably true'.

5.

Mental disturbances is not the same as being a complete 'lunatic'. Thus probably true, [2] is the best answer.

5. Strong Argument, Weak Argument In these types of questions, you have to make decisions, about important issues. While doing so it is desirable to be able to distinguish between 'strong argument' and 'weak argument'. Strong arguments must be both important and directly related to the question. Weak arguments may not be directly related or may be related to trivial aspects of the question and may be of minor importance. Note : It is very important in these questions not to bring in your personal biases and opinions.

Example 1 Plastic Bags should be banned completely. a] Yes : Plastic Bags mostly end up in the garbage dump ending up as cattle food. b] No : If banned completely, products which need to be air proof will suffer. c] Yes : Plastic bags cannot be recycled and therefore are extremely harmful for the environment. 1] only I is strong 3] only I is weak 2] I and II are strong 4] only III is strong

Solution 1. Read the definitions given carefully.

2.

Read the main statement carefully and then look at the arguments put forth. It is essential not to lose focus of the main statement.

3.

Look at I, the argument has to be seen in a broader perspective. It is true that if plastic bags end up as cattle feed it can be harmful but it is not reason enough to ban plastic bags cattle can be prevented from eating plastic bags completely. Thus we car net mark it as a strong argument.

4.

The argument in II is not strong because there are ways and means of packing products which need to be air proof, therefore allowing plastic bags just because packing some products will get affected with the ban is not a strong reason.

5.

III categorically mentions that plastic bags are harmful for the environment because they cannot be recycled. It is directly related to the statement and also a strong reason for banning plastic bags.

6.

Now look at the options, [4] matches with your answer. Mark it off.

Example 2 Children below the age of twelve should not be allowed to watch television as it telecasts a lot of sex and violence. a] Yes : So much of sex and violence have a negative effect on the minds of children who are at an impressionable age. b] No : What is shown on TV is but a reflection of our society, until and unless children get a taste of that, they will grow up to be misfits. c] Yes : They can derive entertainment from sources other than the TV. 1] 1 and II are strong 3] II and III are strong 2] I and III are strong 4] all are weak

Solution 1. Read the main statement and the arguments carefully.

2.

Look at argument I, if sex and violence have a negative effect on children's mind it is a good reason to prohibit them from watching TV.

3.

Keeping children cocooned or sheltered from facing reality leads to their becoming misfits and should therefore be discouraged. This is also a strong reason though it opposes the main statement.

4.

III is not directly related to the topic under discussion and is therefore weak. Thus, the answer is [1].

Note that in this example there are two arguments one opposing and the other favouring the main statement but both are strong.

EXERCISER-5

DIRECTIONS for questions 1 to 5: Each question below is followed by four arguments. You have to classify them into strong and weak arguments.

1.

The private lives of public figures must not be discussed in the press. I] Yes : The press has no right to invade the privacy of an individual, public or otherwise. II] No : Public figures must be used to curiosity about their private lives. III] Yes : The press has a pressing need to be relevant and balanced. IV] No : Public figures are not very sensitive people. 1] Only I is strong. 3] All are weak. 2] All are strong. 4] I and III strong.

2.

Child marriages must be strongly discouraged. I] Yes : This is an evil affecting those who need the most protection. II] No : The custom started with some justification behind it; that must be

understood, and a fine is sufficient. III] Yes : Strong and effective legislation is definitely a deterrent to such social crimes. IV] No : Child marriages are but children's play, after all. 1] I and III strong. 3] All are strong. 2] Only IV is weak. 4] Only III is weak.

3.

God exists. I] Yes : If He did not exist, we would not exist.

II] No : There is no scientific evidence available. III] Yes : An ideal of man exists and that ideal is God. IV] No : Because man controls his. own destiny. 1] Only I is strong. 3] All are weak. 2] Only II is strong. 4] Only III strong.

4.

Voting must be made compulsory. I] Yes : Results will be more representative of the choice of the people.

II] No : Compulsion is no fun. III] Yes : Leaders will govern better. IV] Yes : India is a democracy. 1] 3] I and II strong. Only IV strong. 2] I and III strong. 4] Only 1 strong.

5.

Advertising is a wasteful expenditure. I] Yes : How many people can read ?

II] No : If it was wasteful, then nobody would advertise. III] No : The Advertising industry employs a lot of people. IV] No : Pleasant pictures are worth a thousand words. 1] 3] I and IV are weak. All are weak. 2] Only IV is weak. 4] II and III weak.

ANSWERS KEY

11

22

33

44

5-3

EXPLANATORY ANSWERS
EXERCISE-5

In these types of questions, some amount of elimination can be helpful; but what is important here, is to study the definitions carefully - it is quite difficult to draw the line between a strong and weak argument. 1. Only I is a strong claim for the given statement. 'Generating curiosity' is a vague reason for discussing public figures in newspapers. To be relevant the press need not discuss the private lives of public figures, they can take up other issues. IV is irrelevant. Hence, [1].

2.

You may not agree with statement II but it still makes a strong case for justifying a nominal fine and not a strong punishment. It is important that you don't bring your personal views in answering such questions. Only IV is irrelevant and weak. Hence, [2].

3.

Here II appears strong but look carefully - a thing does not exist because no evidence exists, is a rather spurious argument. Hence, [3].

4.

III and IV are not related to the main statement. II is just a red herring. Note that the main issue here is 'voting' not 'compulsion'. Only I supports the main statement strongly. Hence, [4].

5.

Note that none of the statements point out why or why not advertising is wasteful. Hence, [3]

CHAPTER - 2
CRITICAL REASONING

In these questions a short passage is followed by a question. It is important however to remember that the passage usually consists of certain facts. They may also contain conclusions that can be drawn or inferences that can be made based on the given data. There may be certain assumptions that act as a bridge relating the fact to the

conclusion. Critical reasoning questions call for the student's skill to evaluate and analyze arguments or a given situation. Let us take a few examples :

Example 1 "The streets were deserted. A curfew was imposed in the locality following stray incidents of violence". We may connect these events to come to the conclusion streets are usually deserted when a curfew is imposed.

Example 2 "With the rapid invasion of our lives by the electronic media, sound engineering as a profession has an infinite scope and offers an aspirant several avenues of employment." An assumption which is made in this statement is the rapid advancements in the electronic media has been influential in bringing forth sound engineering as an employment option.

Example 3 Now let us introduce some options. "Night Shyamalan's second movie 'Unbreakable' which again features Bruce Willis, is slated to hit the theatres on Friday. Judging by the immense success of his first movie 'The Sixth Sense', which has a sterling performance by Bruce Willis, this movie is expected to do well at the box office." Which of the following weakens the argument in the passage? 1] 2] 'The Sixth Sense' did not win an Academy Award in spite of its success. 'The Sixth Sense' though a success was not appreciated by all sections of the society. 3] 4] Bruce Willis has had six flops in a row after 'The Sixth Sense'. Shyamalan has been asked to write the script for Spielberg's forthcoming movie.

Tips to solve such questions: 1. Read the passage carefully taking in the facts and the conclusions based on the

facts. For example the facts in this passage are a) Night Shyamalan is directing his second movie 'Unbreakable' after the immense success of his first movie 'The Sixth Sense'. b) Bruce Willis has acted in both the movies.

2.

Look for the conclusion in the passage (note : there may be more than one conclusions). The conclusion in this passage is a) 'Unbreakable' is expected to do well at the box office.

3.

Look out for the assumptions that are taken into consideration to arrive at the conclusion. The assumptions made here are (a) the success of 'Unbreakable' is primarily being based on the huge success of Night Shyamalan's first movie 'The Sixth Sense'. (b) the success of 'Unbreakable' is being based on the excellent performance of Bruce Willis in 'The Sixth Sense'.

The entire process should not take you long though on paper it appears to be so. However it prepares you to tackle any question that is asked of you. Look at the question now - "Which of the following weakens the conclusion above?" Let us now look at the options one by one.

1.

[1] does not weaken the conclusion as we are not concerned whether 'The Sixth Sense' or 'Unbreakable' win Academy Awards are not, a movie may be a disaster at the box office and yet win an Academy Award.

2.

[2] Here again we are not concerned whether it appealed to every section of the society or not but how well it has done at the box office.

3.

[3] If Bruce Willis has had six flops in a row, movie-goers will be wary of watching another movie which stars the same actor. Therefore [3] weakens the conclusion.

4.

[4] If Night Shyamalan is writing a script for Spielberg's movie, Shyamalan's movie will get further fillip, for Spielberg is an ace director, this therefore strengthens the conclusion rather than weakening it.

Example 4 Sindhudurg is a serene beach resort some 400 kms. off Mumbai. There is no direct transportation to Sindhudurg. The nearest railway station to Sindhudurg is a 3 hour bumpy ride in an autorickshaw or a tempo. You can stay in the shanties which provide minimal amenities, however the experience of peace and bliss will make you go back to the place. The Welcome Group of Hotels is planning to build a couple of resorts along the beach with regular bus services to the nearest railway station. They are advertising it as "Come experience the bliss of paradise in Sindhudurg, heaven on earth". Sindudurg will soon cease to be the perfect haven as tourists and vacationers start pouring in making it just another beach resort. Which of the following is an assumption made in the passage? 1] Sindhudurg may soon become just another vacation spot with droves of tourists visiting the place.

2]

Sindudurg has so far not gained publicity for lack of proper transportation.

3]

the Welcome Group of Hotels will attract enough tourists to make Sindhudurg a crowded place.

4]

the Welcome Group of Hotels is going to rob the livelihood of the owners of the shanties.

Solution 1. The facts in this passage are (a) (b) (c) Sindhudurg is a serene beach resort some distance away from Mumbai . The transportation and the lodging facilities in Sindhudurg are minimal. The Welcome Group of Hotels is giving Sindhudurg some publicity by opening a chain of hotels there.

2.

The conclusion that is drawn from the passage is (a) Sindhudurg will soon become like any other crowded beach with the publicity it is getting by the Welcome Group of Hotels.

3.

Now read the question that is asked of you - "Which of the following is an assumption in the passage?"

Note that usually the assumption is something on which the conclusion is based. Now look at the options one by one :

1.

[1] is but a restatement of the conclusion that is drawn and therefore cannot be an assumption.

2.

[2] Note that nowhere is it suggested that it is the lack of transportation that has been the primary factor behind Sindhudurg not being frequented by visitors.

3.

[3] is the correct answer. The conclusion is drawn on the basis of the assumption that once the Welcome Group opens a chain of hotels in Sindhudurg and gives publicity to the place, it will fetch enough visitors to make the place just another crowded beach.

4.

[4] is just a red herring to distract you, at no point does the drift of the passage suggest anything about the shanty owners losing their livelihood.

Example 5 The lives of hundreds of people at risk from kidney disease could be saved by a simple inexpensive test now readily available to the doctors. The multi-banded indicator test gives early warning of kidney damage by detecting the amount of protein in urine along with high risk patients such as those suffering from diabetes or hypertension. Top much may indicate the kidneys are not functioning efficiently and that the patient should be referred to a specialist. Pathologists in India are now encouraging all people to buy home-kits of the same. Which of the following if true would most question the recommendation of the pathologists? 1] It is normal for children to show a high content of protein in urine.

2]

The indicator works only when seen against sunlight.

3] If the test is carried out just after taking a meal it does not always show the correct result.

4] The indicator shows erratic results when there is a slight difference in room temperature.

Solution 1. The facts in the passage are (a) An inexpensive test available with the doctors can detect kidney disorders. (b) Pathologists are encouraging people to buy home kits for carrying out the test.

2.

Note that there is no well-defined conclusion in the passage. However it can be If the test is popularized millions of lives may be saved.

3.

The assumption in the passage is Pathologists trust the efficacy of the test enough to recommend people to buy home kits.

4.

Now read the question carefully, it asks you to find the most important flaw that would question the recommendation of the pathologists.

5.

Checking the options you will see that there is more than one option that questions the efficiency of the test, however our task is to find the glaring one.

1.

It cannot be [1] because the test is not meant only for the children and if there are kids who have kidney disorders they would form a very small section and should not deter pathologists from promoting the product.

2. 3.

It cannot be [2], as checking the indicator against sunlight is not a big deterrent. It cannot be [3], as normally these tests carry specific instructions as to how to conduct the test.

4.

If the test needs to be carried out in an ideal temperature and slight temperature differences show erratic results, it cannot be conducted at home. This, therefore, questions the recommendation of pathologists to use such kits at home, where temperature cannot be regulated. Thus, [4] is the answer.

EXERCISE-6

DIRECTIONS for questions I to 5: Each passage below is followed by a question. Read the passage carefully and answer the question that follows it.

1.

Africa's red-billed oxpeckers had long been thought to aid mammals like the impala by picking ticks off their skin - an example of mutualism. But British biologist Paul Weeks kept oxpeckers away from a group of oxen and found no change in the number of ticks. On other oxen, oxpeckers picked at existing wounds delaying the healing time. A conclusion that can be drawn from the above is 1] Oxpeckers and Impala's have been thought to have a symbibtic relation for long now. 2] 3] 4] Ticks multiply if not scavenged by oxpeckers. Oxpeckers may be carnivorous. All but [3].

2.

Global financial markets froze in the morning New York trade on Wednesday as projections of a Republican sweep of the White House and Congress came unstuck over a recount in the pivotal state of Florida, leaving the presidential election result in balance. The dollar firmed but pared earlier gains against the Euro after the White House race was put on hold. In the event of a clear Republican win in America, which of the following predictions about the dollar are most likely to come true? 1] 2] 3] 4] It will be at an advantage against the Euro. It will rise to some advantage but will eventually slide. It will be stable It will continue to be volatile and unstable.

3.

Inefficiency and its natural corollary, corruption still persists, and nothing symbolises this better than the stacks of files that continue to choke the system. But the department of Income Tax is moving towards computerisation in a bid to speed up processes and improve productivity But are these developments enough? Why not go a step forward and free the individual from the whole

cumbersome process of filing returns? Which of the following would most effectively meet the author's aim? 1] Allowing the individual to make payments through the internet and thus avoiding paperwork 2] Introducing a tax on consumption at the point of sale of goods and abolishing income tax. 3] Introducing a system whereby the government and not the individual is responsible for correct evaluation and payment of income tax. 4] All the above.

4.

Prince William had better watch out. If the proponents of multiculturalism have their way, the most English of British heirs to the throne may have to swerve the bloodline to warmer climes. If the report on "multi-ethnic Britain" by Lord Bhiku Parekh is to be believed that is. The good Lord concludes that the word 'British' itself has racial connotations and the ex-Empire in its twilight years should now be on the lookout for a more inclusive image than the one that it had when the sun never set. Which of the following assumptions is being made by the author in the given passage? 1] 2] Bhiku Parekh is an authority on the history of the British Empire. Lord Parekh's observations of multiculturalism in Britain are echoed in the reality of Britain today. 3] Prince William's marriage to a non-British should dilute the 'English' Bloodline and initiate multiculturalism. 4] Prince William represents an image of Britain that is at odds with Bhiku Parekh's reports.

5.

By the time it's completed in 2005, a statue now slated for construction in Bodh Gaya, India, will be the tallest edifice in the country - and the tallest figure of Buddha on the planet. Artists and engineers in India and Britain are planning the Maitreya Project a meditation park funded by Buddhist donors from all over the world. The centre piece of the 40-acre park will be the 500-foot bronze clad figure, so large that the Statue of Liberty would just reach its arm. Which of the following can be inferred from the passage?

1] 2] 3]

The Maitreya project will take five years to complete. The Statue of Liberty is one of the taller structures in the world. A statue taller than the 500 foot Buddha will not be built anywhere in the world in the next five years time.

4]

Bodh Gaya is a holy place.

ANSWERS KEY

13

21

32

44

5-2

EXPLANATORY ANSWERS

EXERCISE-6
1. [1] is stated in the passage clearly as the subject of research and cannot be the conclusion. [2] is not the answer because the study conducted showed that the number of ticks on impalas did not show any difference when they were not picked by the oxpeckers. If the oxpeckers do not feed on ticks yet pick at the wound it can be concluded that they are carnivorous. Thus [3] is the answer.

2.

"The dollar firmed but pared earlier gains against the Euro". This implies that a Republican sweep would raise the dollar to an advantage against the Euro. Hence, [1].

3.

Read the last sentence carefully. [1] and [3] do not ensure that the individual will not have to file returns. Only [2] does so. Hence, [2].

4.

No such assumption is made by the author, Bhiku Parekh may even be an journalist. Lord Bhiku Parekh's comment is nothing but his personal view, whether that is reflected in present day Britain is not a valid assumption in the passage. [3] is too far fetched and has been forced upon the scheme of things.

Read the first two sentences, it mentions a word of caution for Prince William, therefore he must be thought to represent an 'English' image of Britain. Thus [4] is the answer.

5.

'Now slated for construction' - implies that the construction is yet to begin, therefore it cannot be deduced that the completion will take five years, thus [1] is ruled out. [3] is just an assumption and cannot be inferred from the passage. Though the place chosen for the Maitreya project is Bodh Gaya, we cannot infer from the passage that it is a holy place. If the Statue of Liberty is being compared to one of the tallest structures of the world, it will be a tall structure by itself. Thus only [2] can be inferred.

CHAPTER - 3

VENN DIAGRAMS AND SYLLOGISMS

Venn Diagrams : Venn diagrams are named after a British Mathematician, John Venn who developed the idea of using diagrams to represent sets.

The next question that is likely to be asked is "What are sets?". A set is a well-defined collection of objects. The objects of a set are called its elements or members. A set of animals can include monkeys, leopards, rabbits, jackals, etc. These individual animals are elements of the set of animals.

1.

In Venn diagrams to represent a category that is completely included by the other, the representation is as follows: e.g., All stars twinkle

S = Stars T = Twinkle

2.

To represent a category that is partially included in the other, e.g some stars twinkle, the representation would be as follows

S = Stars T = Twinkle

3.

To represent a category that is completely exclusive of the other, e.g. No stars twinkle, the representation is thus

S = Stars T = Twinkle

4.

If we have an example which says only stars twinkle, it would be represented thus

S = Stars T = Twinkle Only stars twinkle would mean that nothing else twinkles or all that twinkles are stars. Let us take a few more examples

Example 1: Parrots, Sparrows, Squirrel, Finches. Here, Parrots, Sparrows, Finches are -all types of birds whereas Squirrel is not a bird, therefore the first three are elements or members of the set of Birds, but Squirrel falls outside the set of Birds. It would be represented as follows

P = Parrots S = Sparrows F = Finches Sq = Squirrel

Example 3: Mars, Solar System, Universe Here we have three concentric circles, Mars is a part of the solar system which in turn is part of the Universe. The diagrammatic representation is as follows

M = Stars SS = Solar System U = Universe

Example 4: Nib, Ink, Paper Here we have three separate circles which do not touch, meet or overlap. Although we have a mental association of these three items, they have actually no connection with each other. The representation is as follows

N = Nib I = Ink P = Pen

Example 5: Women, Doctors, Anchors

Here we have three overlapping circles with one area in common, because doctors can be women as well as anchors, women can be doctors as well as anchors and similarly anchors. The diagrammatic representation is as follows

EXERC1SE-7

DIRECTIONS for questions 1 to 7: There are five diagrams A, B, C, D, and E given below. In the following questions, three objects /subjects are given. Choose the diagram that best illustrates the relationship between them.

1. 2. 3. 4. 5. 6. 7.

Navy blue, Dark blue, Indigo blue Furniture, Table, Books Lakes, Rain, Water Fathers, Brothers, Husbands T-shirts, Trousers, Skirts Buses, Cars, Vehicles Lemons, citrus fruits, chocolates

ANSWERS KEY

13 61

24 7-4

31

43

55

EXPLANATORY ANSWERS

EXERCISE-7
1. There is an overlapping among the three colours given - the correct option is, therefore, [3].

2.

'Table' belongs to the group 'Furniture', while 'Books' do not. The correct option is [4].

3.

'Lakes' and 'Rain' both belong to the group 'Water' - option [1].

4.

As in question 1, there is an overlapping among 'Fathers', 'Brothers', and 'Husbands'. Some 'Fathers' can be both 'Brothers' and 'Husbands'. Those of you who say "but Fathers MUST fall in the group 'Husbands'", think again. The correct option is [3].

5.

All three are separate items of clothing: option [5].

6.

'Buses' and 'Cars' both belong to the group 'Vehicles'. Option [1].

7.

'Lemons' belongs to the group ' Citrus fruits', while unrelated, hence option [4].

'Chocolates' is entirely

SYLLOGISMS : Syllogism is a mode of thinking in which one reasons from two statements or propositions, called premises to a third statement or a proposition called the conclusion. A premise is a statement that serves as the basis of the argument. Let us elaborate with the help of an example.

Example 1 1. All trees are green. 2. Banyan is a tree. Therefore Banyan is green. The first two statements are premises and the third is the conclusion drawn from the premises. Whether the conclusion is valid or not can be verified with the help of Venn diagrams. The diagrammatic representation will be as follows

B = Banyan T = Tree G = Green As seen from the diagram, if all trees are Green and Banyan is a tree, it is included in the set of Greens and must be Green as well.

Example 2 1. Some professors are serious people. 2. All serious people wear spectacles. Therefore some professors wear

spectacles. The diagrammatic representation will be as follows:

Here the shaded area represents those professors who wear spectacles

Example 3 1. No honey bees buzz. 2. All humming birds buzz. Therefore no humming birds are honey bees. The Venn diagram for this would be as follows:

Here, since Honey Bees fall outside of things that buzz, they cannot be Humming Birds.

Example 4 1. Only boys play football. 2. Lily plays football. Therefore Lily is a boy.

Note : We need to make a distinction between 'only' and 'all'. 'Only' does not mean 'all'. F example when we say only children get toothache we do not mean all children get toothache b that if there is someone who gets toothache it must 'be a child.

Here, since only boys play football, it would mean that all those who play football are therefore, Lily who plays football must also be a boy.

Example 5 1. All dogs bark. 2. Tommy barks. However, here we cannot conclude that Tommy is a dog. All we can conclude

is that Tommy may or may not be a dog. Here, Tommy is another element in the set of Barking things and may not be a dog. The diagrammatic representation would be as follows

Over the years CAT has seen the following kinds of Syllogisms

A. B. C.

Six statements Rows of sentences with three segments in each row Conditional syllogisms

A.

Six statements : In this type of questions six statements are given, these are

followed by four options. The student has to mark the option where the third statement can be concluded from the first two premises or statements. With this type of questions we move into the realm of 'hard-core reasoning'.

Example 1: a] All men lie c] Chou-Chou is not a woman e] Chou-Chou is a woman 1] adc 2] bdc b] All women tell only the truth d] Chou-Chou lies f] Some women lie 3] bfe 4] fde

Now let us examine each of the options one by one and see which are the incorrect sets, and why they are incorrect. Option 1 : adc This is an answer you may mark. However the logic is flawed. Just because 'all men lie' and Chou-Chou lies does not mean that Chou-Chou is not a woman - women could lie too. If the sentence had been "no women lie' then this could have been the

correct answer. A Venn diagram for adc would make it clearer.

There is a possibility that Chou-Chou may be a man. This is shown in the second figure.

Now let us look at option 2 : bdc If women tell only the truth then women is a set inside a bigger set of those who speak the truth, therefore if Chou-Chou lies, Chou-Chou cannot be a woman. Thus [2] is the answer. The same can be represented as follows

Although we have got the answer and ideally need not check the other options, let us do so to check why they are incorrect.

In option 3: bfe 'All woman tell only the truth' and 'some women lie' are contradictory statements and thus nothir an be deduced from contradictory statements. '

In option 4 : fde If some woman lie and Chou-Chou lies we cannot find out whether Chou-Chou is a woman or not. Chou-Chou may be a man, or anything for that matter. The diagram for option [4] would be as follows

Example 2: a] Perry is a nice girl c] All nice girls like sailors e] Perry likes sailors 1] abd 2] dba b] Sherry is a nice girl too d] Perry likes Sherry f] Sherry likes Perry 3] ace 4] abf

In option 1: abd If Perry is a nice girl and Sherry is a nice girl too we cannot logically claim that Perry likes Sherry. They are just two individuals in the set of nice girls.

In option 2 : dba Again if Perry likes Sherry and Sherry is a nice girl, it does not make Perry a nice girl.

In option 3 : If Perry is a nice girl and all nice girls like sailors then Perry who is a girl will like sailors too. The diagrammatic representation would be as follows

In option 4: abf If Perry is a nice girl and Sherry is a nice girl too then we cannot conclude that Sherry

likes Perry. Therefore option 3 is correct.

EXERCISE-8
DIRECTIONS for questions 1 to 7: Choose the correct set of three sentences which make a logical sequence. 1. a) Trout lay eggs. b) Mammals have hair. c) Trout are fish. 1] a c e 2] d d) Platypuses are mammals. e) All fish lay eggs. f) Platypus have hair. 3] a e f 4] c e a

2.

a) A little power corrupts a little. b) Power corrupts. c) A dictator has absolute power. 1] b a f 2] e d f

d) A dictator is absolutely corrupt. e) Corruption is absolute power. f) Absolute power corrupts absolutely. 3] b f c 4]fcd

3.

a) Some men are dancers b) Some men are not dancers c) Sheela may be a dancer 1] a b d 2] d f c

d) Some women are dancers e) Some women are not dancers f) Sheela is a woman 3] d e f 4] e c f

4.

a) Jumbo is a cricketer. b) All cricketers play ball. c) All cricketers bat. 1] a b c 2] a e f

d) Jumbo plays ball. e) Cricketers are fond of chocolates. f) Chocolates affect the teeth 3] e f d 4] b a d

5.

a) Keto likes to look adult. b) Keto is e boy. c) Boys play a lot. 1] a e f 2] a f e

d) Keto likes swimming. e) Keto likes to wear long pants. f) All adults wear long pants. 3] b c d 4] f e a

ANSWERS KEY

14

24

32

44

52

EXPLANATORY ANSWERS
EXERCISE-8 1. If Trout are fish and all fish lay eggs the Trout will lay eggs as well. Hence, [4].

2. Though option [1] might confuse you, you will see that statement (f) does not flow logically from the first two statements. Options [2] and [3] do not make sense. The correct answer is [4].

3. All that this question needs is a little close examination to realise that statements (a) and (b) are irrelevant, and have no third statement to connect to them - therefore you have to make the set of the remaining four statements, and the correct one is option [2] dfc

4. You can immediately use the format 'All Y etc...' to get b a d/ a b d, i.e., option [4].

5. Again sentences (b) and (c) can be discarded - they do not have a third to complete the sequence. With the other four sentences , you can make the sequence 'a f e'. Please note that the same three statements in sequences 'a e f or 'f e a' are correct! not

B. Rows of sentences broken into segments : This is a variation of the same. Here there are three or four rows of sentences broken into three segments, the student has to select the option where the third segment is a logical conclusion from the first two segments.

Example 1 There are three rows of sentences each divided into three segments. Choose the option where the third statement can be logically concluded from the first two. A All men are generous. Peter Pan is generous. Peter Pan is a man. B. No flowers are red. Roses are red. Roses are not flowers. C. Some carpenters are artists. All dancers are artists. Some dancers are

carpenters. 1] only A 2] A and B 3] A and C 4] only B

Let us check all the options one by one.

In option 1: If all men are generous and Peter Pan is generous it does not necessarily imply that Peter Pan is a man. The diagrammatic representation would be as follows:

Since 3 options contain statement A, we can rule out options 1, 2 and 3. Hence, 4 is the answer.

In option 2: If no flowers are red and rose is red then rose cannot be a flower. Note that the world of syllogisms may not conform to the mental associations that we normally assign to things. The diagrammatic representation would be as follows

In option 3: If some carpenters are actors and all dancers are actors they may or may not be carpenters. Thus we cannot logically deduce that some dancers are carpenters. The diagrammatic representation would be thus

Thus the answer is [4].

EXERCISE-9
1. A. HLL is a big company. HLL pays well. Big companies pay well. B. Some robots fly. All birds fly. Some birds are robots. C. All ladies wear cosmetics. Lily is wearing cosmetics. Lily is a lady. 1] only A 4] none of these 2] A and B 3] only C

2.

A. All fried things are bad. Pancakes are fried. Some Pancakes are bad. B. Some men wear tuxedos. Tuxedos are expensive. Men have expensive tastes. C. All butterflies are colourful. Colourful things are pleasant. Butterflies are pleasant. 1] A and B 2] only B 3] A and C 4] only C

3.

A.

Some windows are transparent. All transparent are opaque. Some

windows are opaque. B. All poodles are dogs. All dogs are furry. All poodles are furry. C. Some oval. 1] A and B 2] only A 3] A and C 4] only C tablets are round. Some tablets are oval. Tablets are either round or

4.

A. Some mangoes are sour. All grapes are sour. Some mangoes are grapes. B. Some papers are white. All card boards are brown. Card boards are not paper. C. All men lie. Only liars are welcome. All men are welcome 1] A and B 2] only B 3] A and C 4] only C

5.

A.Some transistors are archaic. Only gramophones are archaic. Some transistors are gramophones

B.

Some flintstones are peanuts. No peanuts are simpsons. All flintstones are simpsons.

No radar are transmitters. All towers are transmitters. No towers are radar. 1] only A 2] A and B 3] only C 4] A and C

ANSWERS KEY

14

24

31

44

54

EXPLANATORY ANSWERS

EXERCISE-9 1. We cannot deduce all big companies pay well by the single instance of HLL, a big company. In B all robots and all birds may be independent sets within the larger set of things that fly. Again the fact that Lily is wearing cosmetics and all ladies wear cosmetics does not make Lily a woman. Thus, none of the conclusions are valid. Our answer is therefore [4]

2.

If all fried things are bad and pancakes are fried then all pancakes would be bad, we cannot say that only some are bad. Again if all men wear tuxedos and tuxedos are expensive then we cannot deduce that only some men have expensive taste. In C if all butterflies are color and all colourful things are pleasant, then butterflies would be pleasant as well. The diagrammatic representation would be

B = Butterflies

C = Colourful P = Pleasant Hence, [4].

3.

In B if all poodles are dogs and all dogs are furry then all poodles should be furry. In C just because some tablets are round and some tablets are oval we cannot deduce that tablets a either round or oval there may be tablets of other shapes too. Thus, the answer is A and B. Hence, [1].

4.

If some mangoes are sour and all grapes are sour we cannot deduce that some mangoes are grapes, they may be separate sets within the larger set of sour things. If some papers are white and all card boards are brown then we cannot deduce that Cardboards are not paper. We may have a possibility as shown in the diagram

P = Paper B = Brown

C = Cardboard W = White

Thus B is not valid. In C if all men are liars and only liars are welcome then all men will be welcome. This can be shown as

Thus the answer is [4].

5.

If some transistors are archaic and only gramophones are archaic, then some transistors have to be gramophones.

G = Gramophone A = Archaic T = Transistor Thus A is valid. B cannot be deduced as they talk of completely different things and there are no common links. In C if no radar are transmitters and all towers are transmitters, then no towers are radar.

Thus both A and C are valid, the answer is therefore [4]!

C.

Conditional Syllogisms : In this type of syllogisms there is a

statement stating a condition followed by pairs of options. The student has to choose the option which satisfies the condition.

Example 1 If the snow continues, we will get stuck. A. B. C. D. The snow continued The snow did not continue We got stuck We did not get stuck 2] DA 3] CA 4] AC

1] AD

The first two options state the opposite of what is stated in the condition, [3] is wrong because it discounts the fact that we can get stuck for other reasons also, [4] is the most logical.

Example 2 Either you are a liar, or you are unfaithful. A. You are a liar B. C. D. You are unfaithful You are not a liar You are not unfaithful 2] CD 3] BC 4] BD

1] AB

The statement implies that you can be one of the two things X or Y; when ysu are X, you are not Y and when you are Y, you are not X. Thus, You are unfaithful would mean that you are not a liar. Thus [3] is the answer.

Example 3 Whenever Raghu sneezes, his nose turns red. A. B. C. D. Raghu's nose did not turn red Raghu sneezed Raghu's nose turned red Raghu did not sneeze 2] AD 3] BD 4] DA

1] CB

Here you may be tempted to mark [1] as the answer, but [1] discounts the fact that Raghus nose can turn red otherwise also. It will become clearer if we explain the same with the help of Venn diagrams. Here the inner circle represents all the times that Raghu sneezes and the outer circle represents the times that his nose turns red. Let us look at option [2] now. The fact that Raghu's nose did not turn red would mean that he did not sneeze because whenever he sneezes his nose turns red. Hence, [2] is the answer.

EXERCISE-10
DIRECTIONS for questions 1 to 5 : A main statement is followed by four statements. Mark t pair of statements which logically follows from the main statement. 1. She scratches her head when she thinks. a] She was thinking. b] She was not thinking. c] She was scratching her head. d] She was not scratching her head. 1] ad b] ca c] ac d] none of these

2.

Either Ramesh is not driving or he is drinking. a] Ramesh is not driving. c] Ramesh is not drinking. 1] ac 2] dc 3] ba b] Ramesh is drinking, d] Ramesh is driving. 4] ab

3.

If he climbs too high his head reels. a] He climbed too high. c] He did not climb too high. 1] ba 2] dc b] His head reeled d] His head did not reel. c] cd 4] ad

4.

Whenever she hears birds sing she sings herself. a] She heard birds sing. c] She did not sing herself. 1] ab 2] cd b] She sang herself. d] She did not hear birds sing. 3] ba 4] all but [3]

5. You will reach Varsha's house only if you take a bus. a] You have reached Varsha's house. c] You haven't reached Varsha's house bus. 1] ba 2] cb 3] bc 4] none of these b] You have taken a bus. d] You havent taken a

ANSWERS KEY

13

21

32

44

54

EXPLANATORY ANSWERS

EXERCISE-10
1. Read the main statement carefully. It implies that her thinking is accompanied by her scratching her head, but not the other way round. Thus [3] is the correct answer.

2.

Note that this is not a simple 'either or statement' as a 'not' is introduced. The statement therefore implies When Ramesh is drinking he is driving and when Ramesh is not drinking he is not driving.. Thus only [1] holds.

3.

This can be represented with the help of Venn diagrams.

As seen from the diagram, his head did not reel would imply that he did not climb too high. Hence, [2].

4.

This is best explained with the help of Venn diagrams.

As seen from the diagram, her hearing birds sing would imply her singing herself. Similarly if she did not sing it would imply she did not hear birds sing. Hence, [4].

5.

Note that you can take the bus to go to other places also, however only a bus will reach you to Varsha's house. Thus none of the options are valid. Hence, [4].

CHAPTER - 4

BINARY LOGIC
This type of question is fairly popular in the CAT examination and is based on Boolean logic. It is best explained by means of an example:

'Colobus' is the name of an island. The inhabitants of this island always answer any question with two sentences, one of which is always true, and the other always false.

Rum, Pum and Posh are the three daughters of the chief of this island. Two of them are married, and one is no.t. You have incurred the chiefs wrath and he has given you two options: identify his unmarried daughter, in which case, you can marry, her. If you cannot, you risk execution. Only Pum has long hair. daughters, these are the answers you get: On questioning the three

Rum Pum Posh

: : :

"I am shorter than Pum. The unmarried woman has long hair" "I am shorter than Rum. Posh is the unmarried one". "The unmarried one is amongst the three of us. I am unmarried."

Who is the unmarried one ? 1] Rum 2] Posh 3] Pum 4] Can't say

The best way of solving this is to use a table, thus: (there is a shortcut, we'll come to that later).

TABLE 1

True Rum: Pum: I am shorter than Pum. Posh is the unmarried one.

False The unmarried woman has long hair. I am shorter than Rum. I am unmarried. *

Posh: The unmarried one is amongst the three of us.

TABLE 2

True Rum: I am shorter than Pum. Pum: Posh: Posh is the unmarried one. I am unmarried.

False The unmarried woman has long hair. I am shorter than Rum. The unmarried one is amongst the three of us.

We have only two possible situations: Table (1), or Table (2) - and of these, only one will be logical. Look at Table (1), and take each sentence one by one.

Rum says: "I am shorter than Pum." We assume this to be true. Hence, Rum is shorter than Pum.

Rum also says "The unmarried woman has long hair," which we assume to be false. Hence, the unmarried woman is not Pum (who has long hair).

Pum says: I am shorter than Rum. But by what Rum has said, this is false.

Hence, her other statement "Posh is the unmarried one" has to be true.

Posh: "The unmarried one is amongst the three of us." HAS to be true. Hence, her second statement "I am unmarried." must be false. But this does not make sense as, in the previous statement we have seen that Posh must be the unmarried one. The assumptions we have made, therefore, do not hold. The sentences marked with an asterisk (*) in Table (1), mark the inconsistencies.

We have now to look at Table (2).

Rum says "I am shorter than Pum", which we assume to be false - i.e., Rum is taller than Pum. "The unmarried woman has long hair," therefore, must be true. That leads us to Pum as the unmarried one.

Pum: "Posh is the unmarried one," must therefore, be false and "I am shorter than Rum."is true - which matches with what we have concluded from Rum's statements. No inconsistencies so far.

Posh: "I am unmarried" has to be false, and "The unmarried one is amongst the three of us" is true - so this is the scenario which is both logical and unique.

Those of you who have caught on, will now say "but you didn't need to go through both the scenarios as described in Tables (1) and (2). All you had to do was look at Posh's statement, see that "the unmarried one is amongst the three of us" HAD to be true, and work from there."

You are right - I did not have to go through both the scenarios - it was for the purpose of illustration. Let me explain, (for the rest of you) how I could have taken a short-cut. We've seen that, "The unmarried one is amongst the three of us" is obviously true. Therefore, Posh is married. Pum's statement "Posh is unmarried" is false, and so she is shorter than Rum, and so on.

So, the lesson is, if there is an obviously true or false statement given, use it to unravel

the answer - you do not have to go through both scenarios. However, if there is no such statement, then the process becomes longer.

EXERCISE-1 1

DIRECTIONS for questions 1 and 2: Read the following carefully.

'Hawa-Hawai' is a tiny country, where the people have peculiar ways of communication - they always answer any question in two sentences, one of which is always true, and the other always false. Unfortunately, you are posted there as Police Chief, and have been called upon to investigate the murder of Pocahontas, the ruling dictator's daughter. You manage to round up three suspects, and after some questioning, they answer as follows:

Simba : "I didn't do it. Aladdin didn't do it." Aladdin : "Simba did it. Baloo didn't do it." Baloo : "Simba did it. I know who did it."

1.

Who murdered Pocahontas ? 1] Simba 2] Aladdin 3] Baloo 4] None of thes

2.

Which of the three lied ? 1] None of them 3] All the three 2] Only Simba 4] Only Baloo

DIRECTIONS for questions 3 to 5: Read the following carefully and answer the questions that follow.

In the village of Scamvihar, all inhabitants always answer any question with two sentences, one of which is always true and the other always false.

While visiting the village, you find Kesri, Deve and Lalloo near the village court. One

of them is wearing a dhoti. Knowing that they were there to resolve a dispute over the ownership of some furniture, you ask them, "who got the chair?". They answer as follows:

Kesri : "I got the chair. Lalloo is wearing a dhoti." Deve : "I am wearing a dhoti. I got the chair." Lalloo : "I got the chair. I am not wearing a dhoti."

3.

Who is wearing a dhoti ? 1] Lalloo 2] Deve 3] Kesri 4] None of them

4.

Who got the chair ? 1] Kesri 2] Can't say 3] Lalloo 4] Deve

On waking up the next morning, you find that your cycle has been stolen. The suspects are the same trio you met the previous day. You question them (knowing that only one of them is guilty), and they reply as follows: Kesri : "Deve did not do it. I did not do it." Deve : "I did not do it. Lalloo did not do it." Lalloo : "I did not do it. I do not know who did it."

5.

Who stole the cycle ? 1] Can't say 2] Lalloo 3] Deve 4] Kesri

ANSWERS

12 23 32 41 5-3

EXPLANATORY ANSWERS
EXERCISE-11

For answers to questions 1 and 2: Going by the shortcut route, we see that Baloo's statements give us the key: Baloo "Simba did it. I know who did it". If the second statement is false, the first does not make any sense - it will have to be false also, which cannot be. Hence, the second statement has to be true, and 'Simba did it' must be false. Then, Simba's first statement is true, and his second - "Aladdin didn't do it." - is false. Thus, Aladdin is the culprit. Hence, [2]. Question 2 is a trick question! Obviously, by the very description of the inhabitants, all of them lie, every time they speak. So, the answer is [3].

For questions 3 to 5: We have to construct a 'truth table' as explained in Chapter 5; as there is no shortcut to be found. We start off by assuming that the first statement of Kesri is true. Then we get the following:

True Kesri : "I got the chair.

False Lalloo is wearing a dhoti."

True Deve : "I am wearing a dhoti.

False I got the chair."

False Lalloo : "I got the chair.

True I am not wearing a dhoti."

As this scenario has no logical inconsistencies, our initial assumption is correct, and the answers we get are, Deve is wearing a dhoti, and Kesri got the chair - i.e., Q.3: option [2], and Q. 4: option [1].

5. Again, assuming the first statement of Kesri to be true, we get the following: True Kesri : "Deve did not do it. False I did not do it."

True Deve : "I did not do it.

False Lalloo did not do it."

We can stop here, as we see a logical inconsistency. As per this scenario, if both 'I did no do it' (Kesri's statement) and 'Lalloo did not do it' are false, that means we have two culprits which is not possible. Hence, we start again, this time assuming that Kesri's first statement is false.

False Kesri : "Deve did not do it.

True I did not do it."

False Deve : "I did not do it.

True Lalloo did not do it."

True Lalloo : "I did not do it.

False I do not know who did it."

This is the correct scenario, and we arrive at option [3], Deve, as the culprit.

TEST-1
DIRECTIONS for questions 1 to 5: Each statement below is followed by four arguments. You have to classify them into strong and weak arguments. Strong arguments must be both important and directly related to the question. Weak arguments may not be directly related or may be related to trivial aspects of the question and may be of minor importance.

1.

Beauty contests are a platform'for the blatant exploitation of women. I. Yes : In such contests, women walk up and down in front of a predominantly male audience II. No : Beauty contests these days test the beauty of the intellect as well. III. Yes : Young girls, dazzled by the glamour and the lure of easy money, are forced to conform to the norms of skimpy attire. IV. Yes: The lack of accepted standards in the industry has lead to a lot of subjectivity in judging the contest.

1] Only III is strong. 3] Only IV is strong. 2.

2] Only I is strong. 4] Only II is strong.

In the allocation of resources, the allocation for defence and poverty eradication, should be equitable. I. No : There are more than 30% of the population below the poverty line.

II. Yes : Poverty is a serious issue, but we also have hostile neighbouring countries. III. No : Make the people strong, and they are the best weapons a country needs. IV. Yes : Democracy means equal treatment for all. 1] All are strong. 3] Only II is strong. 2] Only IV is strong. 4] I and II are strong.

3.

With the present state of infrastructure, we can sustain growth for only two to three years I. No : Past indicators have shown that the growth rate is not dependent on the state of infrastructure. II. No : It is not that the present infrastructure is inadequate; it is merely utilised poorly III. Yes : Poor roads, inadequate power and shoddy ports all contribute toward bottlenecks in industrial growth. IV. Yes : Sustainable growth is a feature of any healthy economy. 1] Only IV is strong. 3] II and III are weak. 2] Only IV is weak. 4] I and III are weak.

4.

Possessiveness usually signals the end of a beautiful relationship. I. Yes : Beauty and possessiveness just don't get along.

II. Yes : When signals flash, you cannot ignore them. III. No : Many a time, the relationship can continue without the beauty. IV. Yes : Possessiveness is not an emotion easily dealt with. 1] Only II is strong. 3] All are weak. 2] Only III is strong. 4] All are strong.

5.

One day you will stop working. Invest wisely today, and you won't stop spending.

I.

Yes : A wise investment today can give one an assured future income.

II. No : It is not necessary that an investment today, however wise, can guarantee 'continued spending' in the future. III. Yes : Someday or the other, everybody stops working. IV. No : Investment is not an area that everybody is comfortable with. 1] II and III are strong. 3] II and IV are strong. 2] I, II and III are strong. 4] I and II are strong.

DIRECTIONS for questions 6 to 10: Choose the diagram that best illustrates the relationship between the three objects / subjects.

6. 7. 8. 9.

Leg, Hand, Face Liquids, Soft-drinks, Coca-Cola Sisters, Mothers, Wives Women, Humans, Living beings

10. Jackals, Lizards, Pigeons

DIRECTIONS for questions 11 to IS: Classify the statements as Fact (F), Inference (I) or Judgment (J), based on the definitions given below. FACT: Something that can be seen or heard, and is capable of being verified. I INFERENCE: A statement that is drawn or concluded from a fact. JUDGMENT: Is an opinion, and implies approval or disapproval

11. I. All in all it's just another brick in the wall. II. Desmond has a barrow in the market place. III. It was a yellow polka-dot bikini. 1] J J F 2] F J F 3] J F I 4] J F F

12. I. The bottle fell from my hands, and-broke into many fragments. II. What a neck it was - willowy, graceful, like the stem of some beautiful flower! III. Regular inhalation of vehicle emissions can lead to a host of respiratory diseases. 1] F I I 2] F J J 3] F J I 4] F F I

13. I. If music be the food of life, play on. II. Rum is a drink that leaves no hangovers. III. Computer operators are in danger of eye-damage. 1] F F F 2] J J J 3] J I I 4] F I I

14. I. You only live twice. II. Life is a roller-coaster. III. This is where the murder happened. 1] J J F 2] J J J 3] F J J 4] I F J

15. I. Marriages are made in Heaven. II. The population problem is getting worse. III. Every seventh person in the world is an Indian. 1] F F F 2] J I F 3] F I J 4] J J J

DIRECTIONS for questions 16 to 21: A passage is followed by statements. Mark [1], if the statement is definitely true Le., it directly follows from the data in the passage. Mark [2], if the statement is probably true but not definitely true in light of the facts in the passage. Mark [3], if the statement is definitely false Le., it contradicts facts in the passage. Mark [4], if the statement is probably false but not definitely false in light of the facts in the passage. Mark [5], if the data is insufficient to make any of the above classifications.

A comparative study of diet and heart disease in seven countries showed that the

death rate from coronary heart disease was highest in countries where the most animal products, including dairy foods were consumed. The Finns, who consumed the most, had the highest death rate from heart disease. Americans were next, except for Seventh-day Adventists, most of whom eat no meat or poultry; they have only half the amount of heart disease that other Americans have. In Japan, where very little fat is eaten, the heart-disease rate is lower than in any other industrialized nation. In one study, it was found that, on diets equally low in fat and cholesterol, persons eating animal protein had higher levels of cholesterol than those fed a diet containing primarily vegetable protein.

16. Americans are healthier than Finns. 17. The death rate of Finns is the highest in the world. 18. Consumption of animal proteins may lead to higher cholesterol levels than the consumption of vegetable proteins. 19. There is a link between consumption of milk, cream, butter, etc., and coronary heart disease. 20. Japan has the lowest incidence of coronary disease, among industrialized nations. 21. Seventh-Day Adventists eat meat and poultry only on the seventh day, i.e., Sunday.

DIRECTIONS for questions 22 to 24: The following questions have six statements each. Choose the set of three statements which make a logical sequence.

22. a) Girls like blue frocks c) Long pants are a sign of adulthood, e) Boys like to be noticed. 1] abd 2] fbc

b) Boys like to look adult d) Girls like to be noticed. f) Boys like to wear long pants 3] bef 4] bef

23. a) All apes have hair. b) All gorillas have hair c) All apes are gorillas 1] dbe 2] cab

d) All primates have hair. e) All apes are primates. f) All gorillas are grey. 3] abc 4] dea

24. a) All silver is metal b) Some silver is metal

d) Some silver is not metal. e) Copper is also metal.

c) All metal is not silver 1] abc 2] aec

f) All metal is copper. 3] ade 4] abd

ANSWERS KEY 11 10 2 19 2 2 3 11 4 20 1 3 2 12 3 21 3 43 13 3 22 4 54 62 74 16 5 83 17 5 94 18 1

14 1 15 2 23 4 24 - 2

TEST 2
DIRECTIONS for questions 1 to 5: Two conclusions are drawn from the main statements. You have to state whether the conclusions are implicit in the main statement.

1.

As he read the letter, Ramirez could not remember why he did not go for the game that Saturday, but the decision has undoubtedly saved his life. I. Ramirezs loss of memory saved his life.

II. What Ramirez did instead of going to the game on Saturday, proved very important for him.

2.

A series of studies showed type A people faced no higher risk of heart problems than anyone else. What emerged was the hostility theory pioneered by Dr. R.R. Williams. I. Other people faced higher risks of heart problems than type A people did.

II. A study on type A people pioneered the hostility theory.

3.

In doubt now, is the partys ability to deal with dissent, thus it is becoming clear that its projected image of being an unconventional party is getting jaded. I. The party has not handled dissent in the best possible manner, leading to a dent in its projected image. II. The passage talks about the Bharatiya Janata Party.

4.

Doctors have also come up against cases of beauty conscious youngsters popping laxatives to keep their bowels empty as often as possible. It achieves nothing more than a false sense of feeling slim and trim. I. Doctors plan to come up with special laxatives for beauty conscious people, who can keep their bowels empty to look slim. II. Doctors are not in favour of people trying to look slim and trim.

5.

But what do entry strategists actually do? We give such an expansive macro eco-political and socio-cultural perspective of the Indian market, that the only thing left for them to do after that, is business: says the director of an entry strategy firm. I. Entry strategists aim to provide exhaustive information about the Indian market. II. Information about the economic scenario in India is provided by entry strategists.

DIRECTIONS for questions 6 to 10: Read the following carefully: An Argument is a statement meant to convince another person about your point of view. An Assertion is a point of view. A Counter - argument contains logic opposing the assertion.

Based on the above definitions, classify each of the given sets of statements into: I. Assertion III. Counter argument II. Supporting reason IV. Irrelevant argument

6.

a) It is difficult to be happy. b) The symptoms of unhappiness are a source of happiness. c) Happiness comes from a lack of want. d) Unhappiness creates a lack of want. 1] I II II II 2] IV I II II 3] I II I II 4] I II IV IV

7.

a) Millions of fishermen complain of catches that have fallen - by half, in some places. b) Industries and governments conspire to bypass laws protecting coastal areas. c) As many as 53 cities in just 3 states discharge sewage directly into the coastal system d) Seas uf waste portend an ecological disaster. 1] I II II IV 2] IV IV II I 3] III II II I 4] II II II I

8.

a) A cheat. c) He's a twisted man. 1] II II I III 2] IV IV II III

b) A liar. d) A friend to be relied on. 3] II III I IV 4] I I II II

9.

a) The difference between good and evil is a fine, dividing line. b) For example, is abortion right or wrong? c) Nature has not devised simplicity. d) Even a small leaf is of infinite complexity. 1] I II I II 2] II I II I 3] II I I III 4] I II III III

10. a) "Yes, I am keen to get married to her. It will be soon." b) He says he plans to get married to a girl he professes to have fallen in love with, while in jail. c) He is a renowned flirt and has broken hearts and promises down the line.

d) Debonair and suave, his dress-sense is not to be faulted. 1] II I I II 2] IV II I IV 3] IV I II III 4] II I III IV

DIRECTIONS for questions 11 to 20: Classify the statements according to the definitions given: A Fact (F) is something that can be seen or heard, and is capable of verification. An Inference (I) is a conclusion following or drawn from, a factual statement. A Judgment (J) is an opinion and implies approval or disapproval.

11. a) Raindrops are falling on my head b) The man is standing on the grass. c) We are the best in painting.

1]FJF

2]FFJ

3] FI J 4]FJJ

12. a) FM radio is a-revolutionary concept. b) Teabags result in a faster method of making tea. c) Tight pants are in. 1] I I J 2] F F F 3] J J J 4] J I F

13. a) White color reflects light. b) Grey color is soothing to the eye. c) Ray Bans are cool. 1] F F J 2] J J F 3] F I J 4] J I F

14. a) It is incredible b) God is great. c) Politics is the last refuge of the scoundrels. 1] F F F 2] J I J 3] J F F 4] J J J

15. a) Catches win matches. b) It's just not cricket. c) Spectators find one-day matches more interesting than test matches. 1] F F J 2] J I J 3] I J I 4] I I J

DIRECTIONS for questions 16 to 20: Read the short passages given below and answer the questions that follow. 16. Competition is good, at least for the consumer. Look at what's happening in the mobile market: state owned giant Mahanagar Telephone Nigam (MTNL) on Friday threw down the gauntlet with introductory cellular rates that are 25% lesser than what the existing private operators charge at present. Following the announcement BPL Mobile has cut their rates by 10% and Orange has reduced its rate by 15%. The general belief being that the number of cellular users will increase with the new tariff rates. Which of the following if true would add credence to the general belief ? 1] Increase in profitability for all three operators. 2] Extension of similar rates for other players like Command which operates in

Calcutta. 3] A study that shows that cellular phone users are price conscious. 4] A study that shows that BPL still thinks that 'it is the best'. 17. Nicknamed mosquito hawks, dragon flies' can change their midair course in a split second. How do they do it? With rubber like vein joints, says Stanislav Gorb, a biologist at Germany's Max Planck Institute. Made of proteins and chitin, dragonfly wings are laced with veins. Where 'they intersect, some veins are joined by an elastic protein called resilin, which makes the wings flexible. "We hope that these rubber like joints will inspire aircraft engineers", says Gorb. Which of the following can put the technology to its optimum use ? 1] The technology can be used in aircraft that ply from Delhi to Bombay one of the busiest sectors. 2] Fighter planes which need maximum maneuverability will derive the maximum advantage 3] The same technology used in aircraft would help their being as light as dragonflies. 4] Both [1] and [2].

DIRECTIONS for questions 18 to 20: Each question has a main statement followed by four statements labelled a, b, c, d. Choose the ordered pair of statements where the first statement implies the second, and the two statements are logically consistent with the main statement.

18. Amar bites his lips when he is nervous. a) Amar is nervous. c) Amar is biting his lips. 1] ad 2] da b) Amar is not nervous. d) Amar is not biting his lips. 3] ac 4] ca

19. If I take a nap in the afternoon, I suffer from insomnia at night. a) I suffered from insomnia at night. b) I took a nap in the afternoon

c) I did not take a nap in the afternoon d) I did not suffer from insomnia. 1] dc 2] ab 3] bd 4] db

20. Unless you break your head over it, you won't get the solution. a) You did not get the solution. c) You broke your head over it. b) You did not break your head over it. d) You got the solution.

1] ac ANSWERS KEY 14 10 4 19 1 2 1 11 2 20 2

2] dc

3] ad

4] none of these.

3 2 12 4

41 13 3

53

62

74 16 3

81 17 2

91 18 3

14 4 15 3

TEST-3
DIRECTIONS for questions 1 to 5: Each question has a set of three statements. Each statement has three segments. Choose the alternative where the third segment in the statement can be logically deduced from the preceding two, but not just one of them. 1. A. All bats are mammals. No mammals are birds. No bats are birds.

B. Some players are losers. All losers are humans. Some players are humans. C. Some balls do not bounce. All ping pongs bounce. Ping pongs are not balls. 1] A and B 2] only A 3] Only B 4] all of these.

2.

A. Monkeys are humans in disguise. Giraffes are not monkeys. Giraffes are not humans in disguise.

B. All servants lie. No liars are truthful. No servants are truthful. C. Some Kulkarnis are Nadkarnis. All Nadkarnis are normal. All Kulkarnis are normal. 1] only A 2] only B 3] B and C 4] only C

3.

A. All storms are natural disasters. Some hurricanes are natural disasters. Some hurricanes are storms.

B. Some rubies are red. Some reds are pinks. Some rubies are pink. C. All violets are shades of blue. All turquoise are shades of blue. Some violets are turquoise. 1] only A 2] A and B 3] only C 4] none of these

4.

A. Some cars run on petrol. Some cars run on diesel. Those which run on diesel do not run on petrol.

B. Only women are allowed to sing. Ravikant sang. Ravikant was a woman. C. Some goats are lambs. All kids are lambs. Some goats are lambs. 1] only A 2] A and B 3] only B 4] none of these.

5.

A. Not all who go to Rome do as the Romans do. Ravi went to Rome. Ravi was a Roman.

B. Some men think they are God. Sushil thinks he is God. Sushil is a man. C. Some flowers dry. All leaves dry. Some flowers do not dry. 1] only A 2] A and C 3] A and B 4] none of these

DIRECTIONS for questions 6 to 10: Each statement below is followed by three arguments. You have to classify them into Strong and Weak arguments. Strong arguments must be both important and directly related to the question. Weak arguments may not be directly related or may be related to trivial aspects of the question, and may be of minor importance.

6.

Access to credit will enable people to work their way out of poverty with dignity and promote their effective participation in social, economic and political activities. I. Yes : Cheap loans provide easy money to many people.

II. No : Easy access to credit will affect the lending records of many banks, in case of default. III. Yes : Many poor people continue in poverty because of a lack of a helping hand, which easy credit will provide. 1] All are weak. 3] Only I is weak. 2] All are strong. 4] Only III is strong.

7.

It is difficult to find a compromise between Science and Ethics, because their scope is different and they are apparently exclusive of each other. I. No : A compromise must be found: if science is divorced from ethics, it can be extremely dangerous for all humanity. II. Yes : A compromise in this field, like all compromises, is dependent totally on the sincerity of both the parties concerned. III. No : While Ethics can exist without Science, Science cannot exist without a modicum of Ethics. They are not exclusive. 1] Only I is strong. 3] Only III is strong. 2] Only II is strong. 4] I and III are strong.

8.

India urgently needs politicians with some social conscience. I. Yes : Today's politicians are concerned only about their own good, and that,

very often, is at the country's expense. II. No : What India really needs is a poverty alleviation program. III. Yes : A lack of social conscience is tantamount to a lack of social responsibility, so essential to any politician. 1] I and III are strong. 3] II and III strong. 2] All are weak. 4] I and II strong.

9.

A hot, dry climate is better for health than a humid, wet climate. I. No : A humid climate leads to healthier skin and system as more water is absorbed. II. Yes : A humid climate leads to perspiration and a disheveled, unattractive appearance. III. No : In hot, dry climates, the use of air-conditioners is excessive, which harms the ozone layer. 1] Only I is weak. 3] Only II is weak. 2] Only I is strong. 4] All are weak.

10. Animals are as important in the eco-system, as humans are. I. No : Man is the most intelligent of all beings.

II. Yes : Each living being is of equal importance in the eco-chain. III. Yes : Any animal has more humanity than a human does. 1] Only II is strong. 3] Only I is strong. 2] Only III is strong. 4] Only I is weak.

DIRECTIONS for questions 11 to 15: Read the following carefully.

An Argument is a statement meant to convince another person about your point of view An Assertion is a point of view. A Counter-argument contains logic opposing the assertion.

Based on the above definitions, classify each of the given sets of statements into: I. Assertion III. Counter argument II. Supporting reason IV. Irrelevant argument

11. a) Such a beautiful place b) There's a garbage dump close by. c) This is a nice area to live in. d) Seems safe, too. 1] I II III IV 2] I I III 3] III I II II 4] II III I II

12. a) Political success is often accidental. b) Mr. Chadboo is one of many politicians who have risen by hard work and commitment. c) From obscurity to fame, sometimes overnight - and all because of Chance.

d) Lady Luck has as her partner, Caprice - and they play havoc with us humans. 1] IV I II II 2] I II III IV 3] I III II IV 4] II I III IV

13. a) An important challenge now is the vast Chinese market. b) China is emerging as a large reservoir of potential to many multinational companies. c) Chinese cuisine has captured the taste of people the world over.

d) The size of the Chinese middle-class is one of the world's largest. 1] I II II II 2] I III I III 3] I II IV II 4] I I II IV

14. a) The dividing line between honesty and dishonesty is very thin b) The dividing line between safety and disaster is very thin. c) One wrong move can mean a lot of pain - sometimes even death. d) 'One little white lie' to save somebody pain - is that a truth or a lie? 1] I II II II 2] I I II II 3] I IV IV IV 4] IV IV IV IV

15. a) The reach of the Internet is limited to very few, technically-oriented people. b) With the Internet, one can explore the frontiers of man's knowledge. c) Millions surf the Internet daily, and thousands join their ranks everyday. d) The Internet opens up a big market for one's business. 1] II II I II 2] III IV II I 3] IV IV II I 4] II IV II I

For the following questions, read the passage and the statements that follow. Classify the statements as: [1] Definitely true in the light of the data given.

[2] Definitely false if the statement contradicts the data in the passage. [3] Probably true if the statement can be inferred as true, but not definitely true. [4] Probably false if the statement can be inferred as false but not definitely false. [5] If the data given is insufficient to make a classification.

The purposive aspects of reflexes, especially in the higher organisms, have given biologists the impression that the organism is a 'machine'... but in their enthusiasm for this comparison, the mechanists have forgotten that every machine is constructed for a given end, and that it pre-supposes a mechanician who has designed and constructed it.

What is the ultimate truth about ourselves? Various answers suggest themselves. We are a bit of stellar matter gone wrong. We are physical machinery - puppets that strut and talk and laugh and die as the hands of time pull the strings. But there is one elementary inescapable answer. We are that which asks the question. ,

16. The school of thought that supposes humans akin to machines, have to admit the existence of a creator of these machines.

17. Biologists think that men are machines; and therefore, immortal.

18. Humans are definitely puppets, manipulated by the hands of time.

19. Many probable answers exist to the question of the ultimate truth about ourselves.

20. The possibility that higher organisms are just physical machinery is advocated by some biologist

21. We are a bit of stellar matter gone wrong.

DIRECTIONS for questions 22 to 25: Given below are statements, each of which is followed by two conclusions. Mark [1], if only conclusion I follows. Mark [2], if only conclusion II follows.

Mark [3], if both conclusions I and II follow. Mark [4], if neither conclusion I nor II follows.

22. Marketing Management is still more art than science, and has slowly yielded to attempts make it more scientific. I. Marketing management has undergone some change.

II. The debate on whether it is art or science, continues.

23. One very important scope of strategic planning is the establishment of the product/market scope of a business. It is here that strategic planning becomes relevant for markets. I. The importance of strategic planning lies in its ability to help establish the market scope of a business. II. Strategic planning involves, among other things, establishing the product or market scope of a business.

24. Throughout 1980, MCI advertised to potential customers in major cities on its network, stressing cost differentials between itself and Bell. I. In 1980, MCI did not advertise to existing customers.

II. Its advertising in 1980 was based on the fact MCI was cheaper than Bell.

25. The experience curve, which encompasses many factors, is a broader concept than the better known learning curve, which refers to the efficiency achieved over a period of time by workers through much repetition. I. The experience curve throws greater light on the efficiency of workers over a period of time. II. A learning curve measures the improvements made by a worker over a period of time, through repetition.

ANSWERS KEY

11 10 1 19 1

2 2 11 4 20 1

3 4 12 3 21 2

43 13 3 22 1

54

64

73 16 3

81 17 5

92 18 4

14 2 15 2 23 3

24 - 4 25 - 4

TEST-4
Given below are statements, each of which is followed by two conclusions. Mark [1], if only conclusion II follows. Mark [2], if both conclusions I and II follow. Mark [3], if only conclusion I follows. Mark [4], if neither conclusion I nor II follows.

1.

Some would argue that the FIIs are coming here, not because of love for India, but because their own markets offer very little returns, as against the rich pickings here. I. There is constant argument as to whether FIIs should enter India, or not.

II. The foreign financial markets are saturated.

2.

A typical multivariate analysis exercise would include using data reduction techniques to obtain a set of groupings, and determining the key strategic variables that define strategic group membership. I. Defining strategic group membership is one of the elements of a multivariate analysis exercises. II. A set of groupings can be obtained by thoroughly defining variables on strategic group membership.

3.

Election time lends itself to making promises and to posturing. The election manifestos of all parties are examples of this syndrome. I. Election manifestos contain promises made by parties.

II. All parties make promises only when it is election time.

4.

Reforms in India have moved at a steady and even pace, unlike in other countries, where they gallop at breakneck speed. I. Galloping reforms are harmful to a country's economy.

II. India's reform process is in some way, different from that in other countries.

5.

With help from its British partner, the firm has sensed the vast potential for CTI

in the Rs.300 crore drive market. I. But for its British partner, the firm could not have sensed the potential of the CTI market. II. The Rs.300 crore drive market will explode with the introduction of CTIs.

Given below is a passage, followed by several possible inferences which can be drawn from the facts stated in the passage. Examine each inference and classify it as follows. Mark [1J, if the inference is 'definitely true', i.e. it clearly follows from the statement of facts given. Mark [2], if the inference is 'probably true', though not 'definitely true', in the light of the facts given. Mark [3], if the inference is 'probably false', though not 'definitely false', in the light of the facts given. Mark [4], if the inference is 'definitely false', i.e. it cannot possibly be drawn from the facts given, or it contradicts the facts . Mark [5], if the data is inadequate, i.e. from the data given, one cannot classify it as above.

All too often, two-wheeler marketing is similar to racing; it is the twists and turns that produce new heroes. The scooter doesn't fit into the picture at all. Typified by the staid, functional workhorse Bajaj Chetak, it has always been a product for the ultraconservative. Bending breezily around corners isn't the stuff of scooter riders, and if caught doing a wheelie, you can safely assume it was involuntary.

6.

Risk takers do well in both racing and two-wheeler marketing.

7.

Scooter riders love doing 'wheelies'.

8.

The scooter is an example of the similarity between racing and two-wheeler marketing.

9.

The Bajaj company produces only staid, functional workhorses.

10. A scooter would, typically, be for a conservative type of person.

DIRECTIONS for questions 11 to 15: Classify the statements as Fact (F), Inference(I), or Judgment (J), as per the definitions given below: A Fact (F) is something that can be seen or heard and is capable of being verifed. An Inference (I) is a conclusion drawn or inferred from a factual statement. A Judgment (J) is an opinion, implying approval or disapproval.

11. a) Good dogs are rare. b) Dogs can be trained c) Cats are smarter than dogs. 1] J J F 2] J I J 3] J I F 4] F 1 J

12. a) Energy can neither be created nor destroyed. b) Every action has an equal and opposite reaction. c) The Earth exerts a great gravitational force. 1] I I I 2] I I F 3] F F I 4] I I J

13. a) The dress could have been a little less showy. b) No man can stop the relentless advance of Time. c) Clouds take on various shapes. 1] F F J 2] I I F 3] J J F 4] J F F

14. a) Death is not the end. b) In a large organisation, one is quite lost. c) The King is dead. 1] F F F 2] J I F 3] J F F 4] J J F

15. a) Mr. Dumpty sat on a wall b) Mr. Dumpty had a great fall. c) He was injured. 1] F J F 2] I F J 3] F I J 4] F F F

DIRECTIONS for questions 16 to 20: There is a statement followed by three assumptions numbered I, II and III. You have to consider which of the assumptions is implicit in the statement.

16. The intervention of government is necessary in all aspects of collective human endeavor. I. Humans are incapable of collective endeavor.

II. The government's intervention is never harmful. III. Collective human endeavor is unlikely to succeed without outside help. 1] Only I is implicit. 3] Only HI is implicit. 2] Only II is implicit. 4] II and III are implicit.

17. Multinationals are expected to change the manufacturing landscape of the country. I. Multinationals are capable of influencing the country's manufacturing sector.

II. Multinationals have a different level of manufacturing from that used in India. III. The multinationals' technology is superior to that used in India. 1] Only III is implicit. 3] II and III are implicit. 2] Only II is implicit. 4] Only I is implicit.

18. A series of studies show that, male fertility across the globe is declining. I. By the turn of the century, males will be infertile.

II. This is nature's way of controlling the population. III. Females are often unjustly blamed for failing to conceive. 1] Only III is implicit. 3] I, II, and III are implicit. 2] II and III are implicit. 4] None are implicit.

19. Production has fallen this year, hence profits will be lower than the last year. I. Lower production leads to lower profits.

II. Prices haven't changed for two years now. III. If profits are lower, prices must have fallen. 1] Only II is implicit. 3] Only III is implicit. 2] Only I is implicit. 4] I and II are implicit.

20. In the spruce little town of Prettimuth, the law-abiding citizens live in neat houses surrounded by manicured gardens - and have manners to match. I. Prettimuth citizens have flowery manners.

II. Prettimuth is a town full of manicursits looking after gardens. III. Prettimuth citizens are well-mannered. 1] Only I is implicit. 3] None of the three are implicit. 2] Only II is implicit. 4] Only III is implicit.

ANSWERS KEY

14 54 95 13 3 17 4

2 3 62 10 1 14 4 18 4

33 73 11 2 15 1 19 2

41 84 12 4 16 3 20 2

TEST-5
DIRECTIONS for questions 1 to 5; A statement is given, followed by three assumptions. Mark those which are implicit in the statement.

1.

Oh, to be a necklace around your graceful, willowy neck, and oh, to nestle against that fragrant, soft, skin! I. Necklaces can inhale aroma.

II. To be a necklace is a desirable thing. III. A graceful, willowy neck needs adornment. 1] Only I is implicit. 3] None are implicit. 2] I and III are implicit. 4] All are implicit.

2.

Thin men get fewer heart attacks than fat men. I. The fatter a man, the less strong his heart is.

II. Thin men are fitter than fat men. III. Thin men live longer than fat men. 1] Only I is implicit. 3] None are implicit. 2] Only II is implicit. 4] Only III is implicit.

3.

Sweaters keep you nice and warm. I. Without sweaters, one always feels cold.

II. Sweaters help you avoid illness. III. If you want to feel warm, you might want to wear a sweater. 1] Only III is implicit. 3] I and III are implicit. 2] I and II are implicit. 4] All are implicit.

4.

Nine out of ten UFO sightings in the U.S.A turn out to be planes. I. One in ten UFO sightings in the U.S.A is actually an UFO.

II. Most people reporting UFO sightings are mistaken. III. Most UFO sightings occur in the U.S.A. 1] Only I is implicit. 3] Only III is implicit. 2] I and II are implicit. 4] Only II is implicit.

5.

Liberalisation has helped most industries; but for the machine tool industry, it has spelt doom. I. Liberalisation has not helped the machine tool industry.

II. Most industries have benefited from liberalisation. III. If it were not for liberalisation, the machine tool industry would have boomed. 1] Only III is implicit. 3] I, II, and III are implicit. 2] I and II are implicit. 4] I and II are implicit.

DIRECTIONS for questions 6 to 11: Choose the set of three statements that make a logical sequence, from the given six.

6.

a) All hope has silver lining. b) All cumulus are clouds. c) In times of despair, there is hope. 1] bfd 2] bed

d) All cumulus have silver linings. e) All despair has silver lining. f) All clouds have silver linings. 3] cae 4] fdb

7.

a) Lawyers wear black. b) Doctors wear white. c) All who wear white are savers. 1] abe 2] acd

d) All who wear black are sober. e) All who wear black are staid. f) All lawyers are staid. 3] dca 4] eaf

8.

a) All fish swim. b) A hen is a bird. c) A hen swims. 1] abc 2] ebf

d) All birds cluck. e) All birds fly. f) A hen flies. 3] dbf 4] def

9.

a) The stars and the sky make up the Universe. b) Life, the stars, and sky belong to each other. c) The Universe is confusing. d) The stars and the sky are universal. e) Life is a part of the Universe. f) Life is confusing.

1] adb

2] ecf

3] abc

4] dab

10. a) All men are dancers. b) All football players are dancers. c) All men are football players. 1] cea 2] eba

d) All dancers are women. e) All football players are men. f) All football players are women. 3] bdf 4] bae

11. a) All mice are rodents. b) Cats eat mice. c) All rodents are grey. 1] dac 2] cad

d) All mice are grey. e) All cats are mice. f) Cats are grey. 3] adc 4] eac

DIRECTIONS for questions 12 to 16: Classify the statements given into Fact, Inference, and Judgment, based on the definitions below: A Fact (F) is something that can be seen or heard and is capable of being verified. An Inference (I) is a conclusion drawn or inferred from a factual statement. A Judgment (J) is an opinion, implying approval or disapproval.

12. a) This is the story of my life. b) I am alive. c) Negroes perform better in athletics than Caucasians do. 1] FIJ 2] JFF 3] JJF 4] JFI

13. a) Judy looks really pretty in a blue dress. b) The problem with this writer is that he writes so much rubbish. c) India should not wage war with Pakistan. 1] FII 2] JJF 3] JIJ 4] JJJ

14. a) I do not blame anybody. b) The film rights have been bought. c) My son is 15 years old. 1] JFF 2] FFF 3] JFI 4] FJJ

15. a) Exposure to radiation leads to cancer. b) The rains this year have been more than in the last year. c) The inflation rate is increasing like a runaway horse. 1] FFF 2] FIJ 3] IIJ 4] JIJ

16. a) We are one of the oldest and most trusted consultant firms. b) This building has both homes and offices . c) Her wonderful laugh filled my ears. 1] FFF 2] IFJ 3] FFJ 4] JFJ

DIRECTIONS for questions 17 to 21: Each statement below is followed by two arguments. Mark [1], If only I is strong. Mark [2], If only II is strong. Mark [3], If both are strong. Mark [4], If both are weak. Strong arguments must be both important and directly related to the question. Weak arguments may not be directly related or may be related to trivial aspects of the question and may be of minor importance.

17. Dams need to be built. I. Yes : They provide nice picnic spots.

II. Yes : They provide water and electricity.

18. Bathing in hot-water springs improves one's health. I. Yes : The minerals in the water help cure gout, arthritis, and a host of other ailments II. No : Studies have shown that the 'improvement' is only perceived, and does not last for more than a few hours.

19. Political instability in this country is now a thing of the past. I. Yes : A new government has been elected.

II. Yes : Nobody likes instability.

20. Use of tobacco leads to cancer. I. Yes : There is extensive data to prove this.

II. No : I have an uncle who smokes, but he does not have cancer.

21. Private firms need to be cleaned up. I. Yes : Public firms are really clean.

II. Yes : The dirt they have is detrimental to their existence.

DIRECTIONS for questions 22 to 24: Refer to the data below.

On an island 'Boola-boola', the inhabitants always answer any question with two sentences - one of which is always true, and the other always false. Read the questions below carefully, and choose the correct answer:

You have been deputized as the commissioner of the island, and discover that smuggling is rampant there. You are determined to catch the smugglers on board their ship, and so you question three suspects, as to when the ship is expected, and what it looks like. This is what they have to say:

Ramesh: "It arrives at 5 p.m. The color of the ship is only blue." Sanjay : "It arrives at 5 p.m. The color of the ship is only black." Rajeev : "I know at what time the ship arrives. Sanjay is lying about the time of arrival."

22. What time does the ship arrive ? 1] 6 p.m. 2] 5 p.m. 3] Can't say 4] Won't arrive

23. What color is the ship ? 1] Can't say 2] Blue 3] Black 4 ] Brown

Suddenly, an epidemic sweeps the island. It is imperative that you locate the person who is the source. The source has to be a person who has been to the mainland within the last 5 days (today is Friday). By careful questioning, you narrow the possibilities

down to three people. This is what they have to say:

Sniffle : "I went to the mainland. It was before Monday." Sneeze : "Sniffle did not go to the mainland. I haven't gone to the mainland in the last 5 days, either. Groan : "Sniffle did not go to the mainland. I am not the source."

24. Who is the source ? 1] Groan 2] Sneeze 3] Sniffle 4] Can't say

ANSWERS KEY

13 52 92 13 4 17 2 21 2

2 3 61 10 3 14 1 18 3 22 2

31 74 11 2 15 3 19 4 23 1

44 82 12 4 16 4 20 1 24 - 3

TEST-6
DIRECTIONS for questions 1 to 5: Each statement below is followed by three arguments, Mark [1], If only I is strong. Mark [2], If only II is strong. Mark [3], If both are strong. Mark [4], If both are weak.

1.

Cigarette smoking should be banned in public places. I. No : Smokers continue to smoke in private anyway. ! II. No : Automobile exhausts are more harmful to health than cigarette smoke is. III. Yes : Even inhalation of smoke from someone else's cigarette, has been shown to ca cancer. 1] Only I is strong. 3] Only II is strong. 2] I, II, and III are strong. 4] Only III is strong.

2.

Synthetic clothing is better for the skin than cotton clothing. I. No : It is known to cause severe skin rashes as it is not made from natural fabric II. Yes : As it is more porous than cotton, it keeps the skin clean and dry. III. No : It contains chemicals that irritate the skin, unlike cotton clothing. 1] Only I is weak. 3] Only III is weak. 2] Only II is weak. 4] All are strong.

3.

Airbags in cars, are more a harmful than a safety feature. I. Yes : The sudden impact as they inflate causes serious injuries.

II. Yes : They do not prevent the driver from hitting the steering wheel. III. No : In most cases, they have prevented impact injuries. 1] Only III is weak. 3] Only I is weak. 2] I and III are strong. 4] II and III are weak

4.

A thing of beauty is a joy forever. I. Yes : I love beautiful things.

II. Yes : Beauty always gives joy. III. No : The concept of beauty changes with time.

1] II and III are strong. 3] All are weak.

2] II and HI are weak. 4] Only II is weak.

5.

The leader of a nation needs to be popular. I. Yes : Democracy is of the people, by the people and for the people.

II. Yes : Popularity polls often reflect election results. III. No : Hitler was a dictator. 1] Only I is strong. 3] Only III is weak. 2] All are weak. 4] Only II is weak.

DIRECTIONS for questions 6 to 10: Choose the set of three sentences that make a logical sequence, from the given six.

a) Akhet is a ship's crew. b) Ashet is a ship's crew. c) All ship's crew are submarine 1] bee 2] bca

d) Akhet and Ashet are friends. e) Ashet is a submarine crew. f) Akhet is a submarine crew, crew. 3] efd 4] bee

7.

a) All men in this coliege smoke. b) Some women in this college, drink. c) Some men in this college drink. d) Some men in this college smoke and drink. e) All men in this college smoke and drink. f) Some women in this college smoke. 1] abf 2] ace 3] acd 4] dca

8.

a) Orang-utans are orange. b) Apes are orang-utans. c) Gorillas are not orange. 1] abc 2] dfa

d) All apes are grey. e) Some apes are gorillas. f) Orang-utans are not apes. 3] dec 4] adf

9.

a) All green is white. b) All white is black. c) All green is black.

d) All black is white. e) All white is green. f) All black is green.

1] cda

2] adc

3] efb

4] deb

10. a) Some cricket players are golfers. b) All golfers are cricket players. c) Susheel is a cricket player. 1] ace 2] bee

d) Sohail is a golfer. e) Susheel is a golfer. f) Sohail is a cricket player. 3] dbc 4] ebc

DIRECTIONS for questions 11 to 14: Each question has a short passage followed by a question. Read the passage carefully and answer the questions that follow.

11. The close contest between two presidential candidates raises questions regarding the nature of democratic representation in the U.S. : Does it undermine or buttress the legitimacy of the presidency? Mr Gore provided the answer though the results were yet to come in : Advance Congratulations to Mr Bush in the event he wins. According to the passage Mr.Gore's statement that 1] the legitimacy of the presidency in the U.S. rests on the unreliable outcome of the election process. 2] Elections in the U.S. corroborate the legitimacy of the presidency. 3] the democratic representation in the states is a facade that hides political maneuvering . 4] there is little scope for a free-and-fair election - even in the most popular democratic nation.

12. Modernity has been misrecognised in India because of the tendency to equate it with technology and with other contemporary artifacts. But the possession of modern technology does not always signify modernity, which has to do with attitudes, especially those that play in social relations. Many forces seethe in urban India today and those who must participate in the social change stand to loose themselves and their colonial attitudes. But in the process, they could gain a new world and perhaps a more just peace. Which of these statements if true would contradict the views of the author as described in the passage above ? 1] People who are modern need not necessarily be computer savvy. 2] We misguidedly tend to equate modernity with the acquisition of modern

scientific products. 3] Unless you master modern technology you cannot participate in social change in today's times. 4] Peace will emanate from shifting age-old paradigms and beliefs.

13. Income tax rates are down to rational levels. The department is also working overtime to project a friendlier face and simplify the collection process. The results have been heartening, with collections up sharply till end September compared with the same period last year. According to the passage, which of the following has not contributed to an increase in tax collections ? 1] A collection process that is less complicated. 2] An improvement in the taxable levels. 3] An effort by the IT department to project a friendlier image. 4] Rationalization of the tax rates.

14. Mr. Gates is a futuristic high technology visionary first and a philanthropic next. The developing world looks to people like him to blend social development and information technology. The synergy can only have a positive fall out on the disadvantaged whom Mr. Gates seeks to help. Mr. Gates strengths in technology are 1] rivaled by his belief on philanthropy. 2] a useful asset to the disadvantaged. 3] likely to prove more useful if combined with social development. 4] [2] and [3].

DIRECTIONS for questions 15 to 20 : Each question has a main statement followed by four statements labelled a, b, c, d. Choose the ordered pair of statements that are logically consistent with the main statement.

15. If I take breakfast, then I do not need to take lunch. a) I took breakfast. c) I needed to take lunch. 1] ab 2] dc b) I did not need to take lunch. d) I did not take breakfast. 3] cd 4] ab and cd

16. Whenever she says no she means yes. a) She said no c) She meant yes 1] ac 2] db b) She did not say no d) She did not mean yes 3] ca 4 ] all but [3].

17. Unless you pull up your socks you will be fired. a) You pulled up your socks. c) You were not fired 1] ab 2] be b) You were fired. d) You did not pull up your socks. 3] ac 4] none of these

18. When she goes to office she carries a bag. a) She is carrying a bag. c) She is not going to office. 1] ba 2] ad b) She is going to office, d) She is not carrying a bag. 3] db 4] none of these

19. You will strike the bull's eye only if you throw the dart from a distance. a) You threw the dart from a distance.

b) You did not strike bull's eye. c) You did not throw the dart from a distance.

d) You struck bull's eye. 1] ad 2] da 3] be 4] all of these.

20. She buys shoes every time she crosses Linking Road. a) She crossed Linking Road. c) She bought shoes. 1] ca 2] ac b) She did not cross Linking Road d) She did not buy shoes. 3] bd 4] all but [1].

ANSWERS KEY

14 52 91

22 61 10 4

32 73 11 2

41 84 12 3

13 2 17 3

14 4 18 1

15 4 19 4

16 4 20 2

EXPLANATORY ANSWERS

TEST-1

1.

Only argument III is directly related to the statement and hence option [1] is the answer

2.

Again, only argument II is directly related to the statement in question, and therefore the correct answer is [3].

3.

All of I, II, and III are directly related to the statement; only statement IV is weak, i.e., option [2].

4.

Here it is clear that none of the arguments are directly related to the statement in question. The answer is [3].

5.

Only I and II are strong arguments as per the definition given; hence, the answer is [4].

6.

These are all unrelated, the answer therefore is [2].

7.

Coca-Cola belongs to the category Soft-drinks, which in turn belongs to Liquids - the figure describing this relationship is [4].

8.

Some Sisters can be Mothers or/and Wives, and some Mothers can belong to either or both the categories Sisters and Wives, and so on. Figure [3] best describes this relationship Hence, [3].

9.

Women belong to the category Humans, who in turn fall into the category Living beings. The correct answer is [4].

10. These are totally unrelated, and the correct answer is therefore [2].

11. Statement I is clearly a Judgment. The second statement is a Fact, which leaves us with options [3] and [4]. Since statement III is clearly not an Inference, the correct option is [4].

12. Since statement I is a Fact and II is a Judgment, we have to now decide whether statement III is an Inference or a Judgment. Matching it with the definition, it appears to be a conclusion drawn from some data - the correct answer is therefore [3].

13. If statement I puzzles you, pass on to II and III, which, going by the definition, appear to be Inferences. Now choosing between options [3] and [4], we can see that the correct option is [3].

14. Statements I and II are clearly Judgments. Choosing between options [1] and [2], since statement III is not a Judgment, the correct option would be [1].

15. The correct option is [2], wherein the first statement is a Judgment, the second an Inference, and the third a Fact.

16. We only have data regarding coronary heart disease, and cannot conclude that, on the whole, Americans are healthier than Finns or vice versa. Hence the answer is Data Insufficient, - option [5].

17. Again, we only have data regarding incidence of heart disease among the Finns. There is absolutely no data regarding the death rate. Hence, we can mark option [5].

18. This is Definitely true, as it is follows directly from the passage. Hence, [1].

19. You may be tempted to mark [1], but look closely, a study shows that there is likely to be a link between consumption of animals products and coronary diseases. It hasn't been proved yet therefore we cannot conclusively say so. Hence, [2].

20. This statement is obviously Definitely true. Hence, [1].

21. This is a clear contradiction of the data given in the passage that most SeventhDay Adventists eat no meat or poultry at all. Hence, option [3] is the correct one.

22. Since there are only two statements referring to Girls, they cannot be part of a logical set - [1] is therefore eliminated. You must understand why bcf is the right answer, and not fbc - read through the statement carefully. Hence, [4].

23. These statements clearly fall into the category All this ..... is that discussed in chapter 3. The correct option is [4].

24. The correct answer here - aec - is best understood when you draw a Venn diagram illustrating it, as follows: METAL

As Copper is also Metal, Metal therefore cannot be exclusively Silver. Hence the answer is [2].

TEST-2

1.

Conclusion I does not follow, as it was not Ramirez loss of memory that saved his life, but the decision not to go for the game, which had. However, since whatever else he did on Saturday proved instrumental in his decision, Conclusion II follows, the correct answer is therefore option [4].

2.

Conclusion I clearly does not follow. Conclusion II docs not follow either - the study did not pioneer the theory, it merely was a conclusion emerging from the

study. We have no data as to when the theory was pioneered. The correct answer is [1].

3.

Conclusion I follows, but II doesnt - don't jump to conclusions, restrict yourself to the data in the main passage. Hence, [2].

4.

Neither of the conclusions follow - hence, option [1] is the right answer.

5.

Both conclusions clearly follow from the information given in the passage. Hence, [3].

6.

As we discussed in Chapter 1 - the best way to figure out the answer is such type of question is to find out the counter argument or the irrelevant statement and use it for eliminating the options which are not correct. In this case, statement a is irrelevant, and therefore [2] is the correct option.

7.

In this question, there is no irrelevant statement to be used for elimination - but d is the assertion, then all the other statements are supporting rea sons. Hence the correct option is [4].

8.

This is a fairly simple one - if statement c is the assertion, then statements a and b are supporting reasons, and d is the counter argument - hence, option [1].

9.

This is an example of two separate assertions and respective supporting reasons. The answer is [1].

10. Statement d is clearly irrelevant, which leaves us options [2] and [4]. Fixing upon statement b as the assertion, we then narrow down the answer to option [4].

11. The first two statements are clearly Facts, and the third statement is a Judgment, as it expresses an opinion. The answer is therefore, [2].

12. Whether it FM is a revolutionary concept or not is very subjective and should

therefore be a Judgement b has to be an inference, one must try both methods to arrive at the inference. Whether tight pants are in or not can be verified by looking around. Hence, [4].

13. Statement c is clearly a Judgment, which leaves options [l] and [3]. We see that the tie-breaker, so to speak, is statement b. As it is not a Fact, we decide on option [3].

14. All these statements are opinions, and the only option that fits is [4].

15. Again, we find the same decision problem as in Q.12, and following the same process, we choose option [3].

16. If the speculation is that the number of people using cellular phones will increase with the cut in tariff rates it goes to show that the consumers are price conscious. Hence, [3].

17. Note the phrase optimum use. Resilin along the joints in th e veins give dragonfly greater maneuverability. Thus only fighter planes can put this aspect to optimum use. Hence, [2].

18. Amars being nervous is indicated by his biting his lips. Thus [3] is valid.

19. Note that my suffering from insomnia does not suggest my taking an afternoon nap, it is the other way round. But the fact that I did not suffer from insomnia implies that I did not take an afternoon nap. Hence, [1].

20. Only [2] is logical.

TEST-3
1. If all bats are mammals and no mammals are birds then that would mean that birds fall outside the set of mammals, thus no bats can be birds. A.

If some players are losers and all losers are humans then some players (those that are losers) will be humans as well. B.

If some balls do not bounce and all ping pongs bounce there is still a possibility that they belong to the set of balls that bounce. Thus C cannot be deduced. The answer therefore is [1].

2.

If all monkeys are humans in disguise and giraffes are not monkeys they may still be humans in disguise. A.

Humans in disguise

Humans in disguise

If all servants lie and no liars are truthful then no servants will be truthful as well.

B.

If some Kulkarnis are Nadkarnis and all Nadkarinis are normal then some Nadkarins (not all) should have been normal. Thus C is not correct. Only B is correct. Hence, [2].

3. 4.

None of the options can be deduced. Hence, [4]. A cannot be deduced, it is a gross assumption. In B only women are allowed to sing, it implies that whoever sings is a women. Thus Ravikant who sings must also be a woman. B.

If some goats are lambs and all kids are lambs it does not follow that some goats are lambs. Thus the answer is [3].

5.

Note that here B seems tempting, but a look at it with the help of Venn diagram says otherwise. B

As seen from the diagram Sushil may or may not be god. C cannot be deduced either. Hence [4] is the answer.

6.

The statement is to do with the help that access to credit will give poor people. Statement I is frivolous, statement II is irrelevant - the answer is option [4], only III is strong.

7.

Statements III is strong and addresses the issue directly, while statement II is irrelevant. The correct answer is option [3].

8.

Again here, as above, II is a weak argument and hence the correct option is [1].

9.

Assuming that statement I is true, it is a strong argument, while statement II is frivolous and III is irrelevant as we are not talking of the atmosphere but of human health. Therefore the correct option is [2].

10. Only statement II is strong, hence option [1] is correct

11. c is an assertion, which statements a and d support. However, statement b is a clear counter argument. The correct answer therefore, is option [4].

12. Either of statements a or d can be assertions. As we do not have the option of d being an assertion, statement a is an assertion, d is irrelevant, and c is clearly a supporting reason. Therefore we choose option [3] as the correct answer.

13. At first glance, this question might be confusing. But once we see that statement c is clearly irrelevant, then the option is clear - [3].

14. This is a case of two separate assertions with respective supporting reasons. Statement c supports assertion b, and statement d supports assertion a. The answer is option [2].

15. Statement a could be an assertion, with c as its counter argument (or vice versa) but that option is not available. Therefore the correct answer is option [2].

16. Going by the data given in the passage, this statement will be Probably true; as it is not a direct part of the passage, we cannot say that it is Definitely true. Hence, [3].

17. We have absolutely no data in the passage to indicate that biologists think that men are machines; and therefore, immortal - hence we mark [5] - Data Insufficient.

18. Why is this marked probably false? Please note that the sentence is, Humans are definitely puppets, manipulated by the hands of time - it is probably false that they are definitely puppets, while it is true that they are probably puppets. Hence, [4].

19. This, according to the passage is Definitely true. Hence, [1].

20. This, according to the passage is Definitely true. Hence, [1].

21. This, according to the passage is Probably true as the statements is a suggestion, but not Definitely true. Hence, [2].

22. Clearly, conclusion I follows. There is no mention of any debate in the statement, and hence we mark option [1] as the answer.

23. Both the conclusions can be drawn from the statement, and hence we mark option [3] as the answer.

24. Conclusion I is a clear contradiction to what is in the statement, conclusion II although likely to be true cannot be definitely taken as true. Hence, [4].

25. Neither of the conclusions follow. Please note carefully the difference between what is there in the statement and what is given in conclusion II - and why it cannot be concluded.

Note: Although the set of questions is similar to those in implicit statements, there is a subtle difference. In these, we have state to whether the conclusions follow or not. while in the former we have to state whether the statements mentioned are implied in the passage or not.

TEST- 4

1.

Clearly, neither of the conclusions can be drawn from the statement given. Do not get confused as to the second conclusion - just because foreign markets offer very little returns does not mean that they are saturated. The correct option is [4].

2.

This statement looks confusing, doesn't it! Just eliminate the adjectives, to get the gist of the sentence, thus: A typical (multivariate analysis) exercise would include using (data reduction) techniques to obtain a set of groupings, and determining the (key strategic) variables that define (strategic group) membership. Read the sentence excluding the parts within the brackets. You will see that only conclusion follows ... Still confused? Take a deep breath and read it again till it makes sense! Hence, [3].

3.

Only conclusion I clearly follows. The answer is option [3].

4.

Clearly, conclusion I is not something we can infer from the data given. Conclusion II is, however, clear enough - the answer is, therefore, option [1].

5.

We cannot say that, without help from its British partner, the firm could not have sensed the potential. The British partner helped, but there is nothing to say that it was indispensable. Conclusion II clearly does not follow. Hence, the answer is

option [4].

6.

Given the data above, we cannot say that it is definitely true but it is probably true It do not fall into any of the other categories, hence we mark option [2].

7.

Now, this is either definitely false or probably false To decide between the two use this thunb rule: if the statement clearly contradicts data given, then it is definitely false. In this case, we see that the contradiction is not direct. Therefore we mark option [3].

8.

Now this is a statement that can be marked as definitely false because it is a clear contradiction of practically the entire passage. Hence, [4].

9.

We have no data whatsoever on the product range of the Bajaj company. This is an example of how to decide whether data insufficient should be marked as an option. Hence, [5]

10. Again, this is a paraphrase of practically the entire paragraph. The answer is option [1] Definitely true.

11. Statements a and c are clearly Judgments. The only option that matches this classification is [2].

12. Statement c is a Judgment - the judgmental aspect of the statement comes from the adjective great, which then turns the statement into an opinion. The answer therefore, is option [4].

13. Both statements a and b are Judgments. Hence we mark option [3]. But please realsie why statement c is a Fact. As per the defin ition given, this is something that can be seen and is capable of verification. Remember, your task is not to judge whether a statement is true or not. Your task is to classify as per the definition.

14. Again, statements a and b are Judgments. The answer is therefore option [4].

15. Statements a and c are Facts. We still have to choose between options [1] and [4]. The word great in statement b turns it into a Judgment, as great is an opinion. Therefore we choose option [1].

16. It is clear here that neither I nor II are implicit. III, however, is. These questions are similar to questions 1 to 5 above, which conclusion follows? Hence, [3].

17. You might think that both I and II are implicit. But we do not have that option. We then see that I is more clearly implicit. Thus by a process of elimination, we choose option [4].

18. None of the three statements can be said to be implicit. The answer is therefore option [4].

19. The sentence, phrased another way, is nothing but I. II and III are not implicit. The answer is option [2] - Only I is implicit.

20. This question needs just a little thought. Do distinguish between flowery manners, which is not implicit, and well-mannered, which is. The answer is [4] - Only III is implicit.

TEST-5

1.

Clearly, none of the three are implicit assumptions. Hence we mark option [3], None are implicit.

2.

None of the three are implicit. It is not necessary that fewer heart attacks means stronger hearts. It is also not necessary that thin men live longer than fat men there could be other causes of death, than heart attacks. Hence, the answer is option [3].

3.

Just because sweaters keep you warm does not mean that without sweaters, one

feels cold. However, statement III is implicit. The answer is option [1], only III is implicit.

4.

This is a slightly tricky one. Note that, if nine out of ten UFO sightings in the U.S.A turn out to be planes, it is not necessary that the tenth sighting is that of a UFO - it is also not implied that most UFO sightings happen only in the U.S.A. Therefore, the answer is clearly, option [4] only II is implicit.

5.

Both I and II are clearly implicit; but III is not. We know that liberalisation has not helped the machine tool industry; that does not mean that If it were not for liberalisation, the machine tool industry would have boomed. Hence, the answer is option [2], I and II are implicit.

6.

This question is of the type All this ... is that as described earlier. As soon as we see the statement All cumulus are clouds, then we look for the quality that cloud possesses All clouds have silver linings. The third statement then obviously has to be All cumulus have silver linings - option [1].

This is also of the type All this.... are that, and the correct answer is [4], e a f.

8.

Again, the type All this is that. Once one sees the statement A hen is a bird, we look for the quality that birds have All birds fly . The third statement then would be A hen flies - option [2].

The correct answer will be option [2]. Do not use your knowledge of the world to influence the selection of the answer, but please check which set makes a logical sequence.

10. Again, this is of the type 'All this ... are that', and the correct answer is option [3]. You can draw a Venn diagram to verify your answer, as follows:

11. Yes, yet another question of the type All this... are that and the correct answer is option [2].

12. Statement a is clearly an opinion, and hence a Judgment. This does not eliminate many options, as we are still left with [2], [3], and [4]. The second statement can be seen, heard, and is capable of being verified, and hence is clearly a Fact. This leaves us with options [2] and [4]; we now have to choose between Fact and Inference, for the third statement. As this statement does not contain factual data, but appears to be a conclusion drawn from some data, we classify it as an Inference, choosing option [4].

13. The first and second statements are clearly Judgments. We are left with options [2] and [4], i.e., we have to decide whether the third statement is a Fact or a Judgment. As it is clearly not a Fact (it could be an Inference, but we do not have to really bother about that - we have to just choose between the options available), we choose option [4].

14. Again, the first statement is, a Judgment. This gives us options [1] and [3]. As the second statement is classified as a Fact in both the options, we move on to the third statement, which is clearly a Fact, and choose option [1].

15. This is a question where one cannot use the technique of elimination, as we have done in the previous questions. The only clear lead we have is that the third statement is a Judgement - which still leaves us with options [2], [3], and [4]. As both the 1st and 2nd statements are not Judgments, but appear to be conclusions drawn from factual data, we mark them as Inferences, option [3].

16. As statement 1 is clearly an opinion, and hence a Judgment, we mark option [4] as the correct one, as it is the only option which classifies the first statement as a Judgment.

17. Argument I is frivolous, and hence a weak one. Argument II however, provides a strong case as to why dams need to be built. Hence, the answer is option [2].

18. Both the arguments are related directly to the statement in question, and are hence strong. The answer is, therefore, option [3],

19. As neither of the arguments address the issue of political instability being a thing of the past - just because a new government is elected does not mean that instability is now over - they are both weak, and the answer is [4].

20. Clearly, Argument I is a strong one, addressing the issue directly. But Argument II does not address the main issue of, in general, use of tobacco leading to cancer. Hence, the answer is option [1] - only Argument I is strong.

21. Argument I does not have anything to do with the issue the statement addresses, and hence is weak. Argument II, however, is a strong one, and hence the correct option is [2].

22. There are two short - cuts you can take, to arrive at the answers. Let me explain both: Ramesh : It arrives at 5 p.m. The color of the ship is only blue. Sanjay : It arrives at 5 p.m. The color of the ship is only black. Now, the first statements of both Ramesh and Sanjay have to be true. If they are false, then we get contradictory answers for color of the ship, which is not possible - i.e., colour of the ship is only blue will be true, and color of the ship is only black will also be true - this is not possible. Therefore, we get time of arrival as 5 p.m., and we cannot say what the colour of the ship is. This is the easier and quicker of the two methods. Thus the answer for 22 is [2].

Short-cut II:

Rajeev : I know at what time the ship arrives. Sanjay is lying about the time of arrival.

23. Looking at Rajeevs statements, the first one has to be true. If it is false, then the second one does not make any sense - Rajeev cannot tell if Sanjay is lying, if he does not know at what time the ship arrives. So, since his first statement is true and the second false, Sanjay is telling the truth about the time of arrival - which is 5 p.m., and since the statements about the color are contradictory, we cannot say what color the ship is. Thus the answer for 23 is [1].

24. Here again, there is a short-cut to unravel the statements: Look at Sniffles statements. Sniffle: I went to the mainland. It was before Monday. Statement I has to be true: if it is false, then It was before Monday does not make any sense. Hence, Sniffle went to the mainland after Monday and is the source. Hence, [3].

TEST-6

1.

This type of question is often fairly difficult to figure out. A thumb-rule to solve them more accurately: First, underline the main theme of the sentence. Then, check if the argument directly addresses the main theme or not. If it doesnt, it is weak. To illustrate, Cigarette smoking should be banned in public places. The underlined portion is really the main theme - whether cigarette smoking should be banned in public or not. Then we see that Arguments I and II do not address this part of the sentence at all, while Argument III does, giving a reason why smoking should be banned in public places. Hence the answer is [4], only III is strong.

2.

Here the main theme is not the absolute merit of synthetic clothing; it is the

relative merit of synthetic clothing as compared to cotton clothing. All the three arguments therefore, are strong , and the answer is option [4].

3.

Again, here the theme is, whether airbags are more harmful than beneficial. Thus arguments I and III are strong. Argument II does not talk about whether the harm is more than the benefit. Hence it is weak. The answer is option [2].

4.

Here, argument I is obviously irrelevant and not connected at all to the statement, hence the correct option is [1], II and III are strong.

5.

In this case, none of the statements are directly connected to the statement, and hence, the answer is option [2], all are weak.

6.

The first clue is that statement c has to be a part of the logical sequence, as no logical sequence can exist without it. But that eliminates only one option. The second clue is that, statement c has to be either the first or the second statement. That leaves us with options [1] and [2]. Then we can see that option [1] - 'bee' is the answer.

7.

This question can easily be understood with the help of Venn diagrams. The correct answer, option [3] - a] All men in this college smoke, c] Some men in this college drink, d] Some men in this college smoke and drink - illustrates as follows: Men (all smokers)

Using similar Venn diagrams, we see how none of the other options are logically correct.

8.

This is best explained with the help of venn diagrams. Thus the answer is [4].

9.

This is a question where at least five logical sets can be formed cda, abc, def, ecb and fad. The one that is present in the options is cda, option [1].

10. Again, this is of the type 'All this ... are that'. The answer is option [4] 'ebc'.

11. The passage says the Mr. Gores statement answers the first question, i.e. Yes, it supports or No, it doesnt. Hence, [2] which is the only option that answers the question. Note that Gores statement, which expresses wil lingness to go by the electoral outcome is a positive endorsement of the process. Hence, [2].

12. The author speaks of attitudes being the prime factor in inducing social change technology cannot suffice alone. All other options do not contradict the author's views. Hence, [3]

13. Tax rates, not taxable limits have improved. Hence, [2].

14. Both [2] and [3] are true and follow from the passage. Hence, [4].

15. The sentence implies that my taking breakfast means my not needing to take lunch, thus both [1] and [3] are true. Hence, [4].

16. This is best described with the help of Venn diagrams.

As seen from the diagram She said no would mean her meaning yes and She did not mean yes would imply her not saying no. Hence, [4J. 17. Note the word unless. The statement implies that if you pull up your socks you wont be fired. Hence, [3]. 18. Note that she may carry a bag at other times also, thus her carrying a bag does not imply her going to office. Hence, [1]. 19. Note the word only. Thus striking the bulls eye would imply having thrown the dart from a distance. All the pairs are logical. Hence, [4]. 20. Note that she can buy shoes at other times also, thus [3] does nut hold. Only [2] is logical, Hence, [2].

Anda mungkin juga menyukai